You are on page 1of 85

Total Marks : 200

Test-4 (Subject)
( INSTA Prelims Test Series 2021 )

1. Consider the following statements regarding Indian National Congress


1. The first session of Indian National Congress was held at Bombay in 1885.
2. Dadabhai Naoroji was its first president.

Which of the statements given above is/are correct?


A. 1 only
B. 2 only
C. Both 1 and 2
D. Neither 1 nor 2

Correct Answer : A

Answer Justification :

The Indian National Congress (1885)

Allan Octavian Hume, a retired civil servant in the British Government took the
initiative to form an all-India organization. Thus, the Indian National Congress was
founded and its first session was held at Bombay in 1885. Hence, statement 1 is
correct.

W.C. Banerjee was its first president. Hence, statement 2 is incorrect. It was attended
by 72 delegates from all over India. Persons attending the session belonged to different
religious faiths. They discussed the problems of all the Indians irrespective of their religion,
caste, language and regions.

2. Which of the following were the major decisions/outcomes of Lahore Session, 1929?
1. Boycotting the first Round Table Conference
2. Congress Working Committee was authorized to launch a programme of civil disobedience
including non-payment of taxes.
3. Congress directed its members to resign from legislative councils

Select the correct answer using the code given below


A. 1 and 3 only
B. 1 and 2 only
C. 2 and 3 only
D. 1, 2 and 3

Correct Answer : D

Answer Justification :

www.insightsactivelearn.com 1
Total Marks : 200
Test-4 (Subject)
( INSTA Prelims Test Series 2021 )

Lahore Congress and Purna Swaraj

Jawaharlal Nehru, who had done more than anyone else to popularize the concept of Purna
Swaraj, was nominated the president for the Lahore session of the Congress (December 1929)
mainly due to Gandhi’s backing (15 out of 18 Provincial Congress Committees had opposed
Nehru).

The following major decisions were taken at the Lahore session.

The Round Table Conference was to be boycotted. Hence Statement 1 is correct.


Complete independence was declared as the aim of the Congress.
Congress Working Committee was authorised to launch a programme of civil
disobedience including non-payment of taxes and all members of legislatures
were asked to resign their seats. Hence Statement 2 is correct.
January 26, 1930 was fixed as the first Independence (Swarajya) Day, to be celebrated
everywhere.

3. Madari Pasi, a famous peasant leader was associated with

A. Bardoli Satyagraha
B. Indigo Revolt (1859 -60)
C. Eka Movement
D. Tebhaga Movement

Correct Answer : C

Answer Justification :

Eka Movement or Unity Movement was a peasant movement which surfaced in


Hardoi, Bahraich and Sitapur during the end of 1921 by Madari Pasi an offshoot of
Non Cooperation Movement.

Hence, option (c) is correct.

The initial thrust was given by the leaders of Congress and Khilafat movement.

The main reasons for the movement were:

High rent, which was generally higher than 50% of recorded rent in some areas.
Oppression by thekedars who were entrusted to collect rent and
Practice of share rent also contributed to this movement.

www.insightsactivelearn.com 2
Total Marks : 200
Test-4 (Subject)
( INSTA Prelims Test Series 2021 )

By March 1922, severe repression by authorities brought the movement to an end.

4. Consider the following statements regarding Bal Gangadhar Tilak


1. Bal Gangadhar Tilak was popularly known as ‘Lokamanya’.
2. He declared “Swaraj is my birth-right and I will have it.”
3. He founded the Indian Home Rule League in the US in 1916.

Which of the statements given above is/are correct?


A. 1 and 2 only
B. 1 and 3 only
C. 2 and 3 only
D. 1, 2 and 3

Correct Answer : A

Answer Justification :

Leaders of the Extremists

The extremists were led by Bala Gangadhar Tilak, Lala Lajpat Rai, Bipinchandra Pal and
Aurobindo Ghosh.

Bal Gangadhar Tilak is regarded as the real founder of the popular anti-British movement in
India. He was known as ‘Lokamanya’. He attacked the British through his weeklies The
Mahratta and the Kesari. He was jailed twice by the British for his nationalist activities and
in 1908 was deported to Mandalay for six years. He set up the Home Rule League in 1916
at Poona and declared “Swaraj is my birth-right and I will have it.”.

Hence, statement 1 and statement 2 are correct.

Lala Lajpat Rai is popularly known as the ‘Lion of Punjab’. He played an important role in
the Swadeshi Movement. He founded the Indian Home Rule League in the US in 1916.
Hence, statement 3 is incorrect. He was deported to Mandalay on the grounds of sedition.
He received fatal injuries while leading a procession against the Simon Commission and died
on November 17, 1928. Bipan Chandra Pal began his career as a moderate and turned an
extremist. He played an important role in the Swadeshi Movement. He preached nationalism
through the nook and corner of India by his powerful speeches and writings.

www.insightsactivelearn.com 3
Total Marks : 200
Test-4 (Subject)
( INSTA Prelims Test Series 2021 )

5. AUDFs01, recently seen in news, is

A. Artificial Intelligence based Audio system


B. Recently discovered Galaxy
C. Malware
D. Covid 19 vaccine developed by Japan

Correct Answer : B

Answer Justification :

As a landmark achievement in Space missions, Indian Astronomers have discovered one of the
farthest Star galaxies in the universe.

India’s first Multi-Wavelength Space Observatory "AstroSat" has detected extreme-UV


light from a galaxy located 9.3 billion light-years away from Earth. The galaxy called
AUDFs01 was discovered by a team of Astronomers led by Dr Kanak Saha from the
Inter-University Centre for Astronomy and Astrophysics (IUCAA), Pune.

Hence, option (c) correct.

https://pib.gov.in/PressReleasePage.aspx?PRID=1650396

6. Which of the following was/were the main demands of Moderates leaders of Indian National Congress (INC)?
1. Abolition of salt tax and sugar duty
2. Separation of the judiciary from the executive
3. Freedom of speech and expression and freedom to form Associations
4. Reduction of spending on army

Select the correct answer using the code given below:


A. 1, 2 and 3 only
B. 2, 3 and 4 only
C. 1, 2 and 4 only
D. 1, 2, 3 and 4

Correct Answer : D

Answer Justification :

All the statements given are correct.

www.insightsactivelearn.com 4
Total Marks : 200
Test-4 (Subject)
( INSTA Prelims Test Series 2021 )

Main Demands of Moderates

_ Expansion and reform of legislative councils.

_ Greater opportunities for Indians in higher posts by holding the ICS examination
simultaneously in England and in India.

_ Separation of the judiciary from the executive.

_ More powers for the local bodies.

_ Reduction of land revenue and protection of peasants from unjust landlords.

_ Abolition of salt tax and sugar duty.

_ Reduction of spending on army.

_ Freedom of speech and expression and freedom to form Associations

7. Consider the following statements regarding Gandhi’s Eleven Demands


1. Abolish land revenue
2. Carry out reforms in Criminal Investigation Department (CID).
3. Reserve coastal shipping for Indians.

Which of the statements given above is/are correct?


A. 2 only
B. 1 only
C. 2 and 3 only
D. 1 and 3 only

Correct Answer : C

Answer Justification :

Solution: C

Gandhi’s Eleven Demands

To carry forward the mandate given by the Lahore Congress, Gandhi presented eleven
demands to the government and gave an ultimatum of January 31, 1930 to accept or reject
these demands. The demands were as follows.

Issues of General Interest

1. Reduce expenditure on Army and civil services by 50 per cent.

www.insightsactivelearn.com 5
Total Marks : 200
Test-4 (Subject)
( INSTA Prelims Test Series 2021 )

2. Introduce total prohibition.

3. Carry out reforms in Criminal Investigation Department (CID). Hence Statement


2 is correct.

4. Change Arms Act allowing popular control of issue of firearms licences.

5. Release political prisoners.

6. Accept Postal Reservation Bill.

Specific Bourgeois Demands

7. Reduce rupee-sterling exchange ratio to 1s 4d

8. Introduce textile protection.

9. Reserve coastal shipping for Indians. Hence Statement 3 is correct.

Specific Peasant Demands

10. Reduce land revenue by 50 per cent. Hence Statement 1 is incorrect.

11. Abolish salt tax and government’s salt monopoly.

8. Consider the following statements regarding the Deccan peasants uprising, 1875:
1. It was directed mainly against the excesses of the Marwari and Gujarati money lenders.
2. The peasants attacked the moneylender’s houses, shops and burnt the documents.
3. The Agriculturists Relief Act of 1879 was passed by the government to protect the interest of
peasants.

Which of the statements given above is/are correct?


A. 1 and 2 only
B. 2 and 3 only
C. 1 and 3 only
D. 1, 2 and 3

Correct Answer : D

Answer Justification :

All the statements given above are correct.

The Deccan peasants uprising, 1875 was directed mainly against the excesses of the

www.insightsactivelearn.com 6
Total Marks : 200
Test-4 (Subject)
( INSTA Prelims Test Series 2021 )

Marwari and Gujarati money lenders.

The movement began at Supa, a large village in Poona (present-day Pune) district. It was a
market centre where many shopkeepers and moneylenders lived.

On 12 May1875, ryots from surrounding rural areas gathered and attacked the
shopkeepers, demanding their bahi khatas (account books) and debt bonds. They
burnt the khatas, looted grain shops, and in some cases set fire to the houses of
sahukar.

The Government appointed the Deccan Riots Commission to investigate into the
causes of the uprising.

The Agriculturists Relief Act of 1879 was passed by the government to protect the
interest of peasants. It put restrictions on the operations of the peasants land and
prohibited imprisonment of the peasants of the Deccan for failure to repay debts to
the moneylenders.

9. Consider the following statements regarding the Extremist leaders of Indian National Congress (INC):
1. The social base was zamindars and upper middle classes in towns.
2. They believed that political connections with Britain would perpetuate British exploitation of
India.
3. They had immense faith in the capacity of masses to participate and to make sacrifices.

Which of the statements given above is/are correct?


A. 1 and 2 only
B. 1 and 3 only
C. 2 and 3 only
D. 1, 2 and 3

Correct Answer : C

Answer Justification :

Moderates

1. Social base—zamindars and upper middle classes in towns. Hence, statement 1 is


incorrect.

2. Ideological inspiration— western liberal thought and European history.

3. Believed in England’s providential mission in India.

4. Believed political connections with Britain to be in India’s social, political and cultural
interests.

www.insightsactivelearn.com 7
Total Marks : 200
Test-4 (Subject)
( INSTA Prelims Test Series 2021 )

5. Professed loyalty to the British Crown.

6. Believed that the movement should be limited to middle class intelligentsia; masses not yet
ready for participation in political work.

7. Demanded constitutional reforms and share for Indians in services.

8. Insisted on the use of constitutional methods only.

9. They were patriots and did not play the role of a comprador class.

Extremists

1. Social base—educated middle and lower middle classes in towns.

2. Ideological inspiration—Indian history, cultural heritage and Hindu traditional


symbols.

3. Rejected ‘providential mission theory’ as an illusion.

4. Believed that political connections with Britain would perpetuate British exploitation of
India.

5. Believed that the British Crown was unworthy of claiming Indian loyalty.

6. Had immense faith in the capacity of masses to participate and to make sacrifices.

7. Demanded swaraj as the panacea for India's ills.

8. Did not hesitate to use extraconstitutional methods like boycott and passive resistance to
achieve their objectives.

9. They were patriots who made sacrifices for the sake of the country.

Methods of Moderates

The Moderates had total faith in the British sense of justice and fair play. They were loyal to
the British. They looked to England for inspiration and guidance. The Moderates used
petitions, resolutions, meetings, leaflets and pamphlets, memorandum and delegations to
present their demands. They confined their political activities to the educated classes only.
Their aim was to attain political rights and self-government stage by stage.

In the beginning, the British Government welcomed the birth of the Indian National Congress.
In 1886, Governor General Lord Dufferin gave a tea garden party for the Congress members in
Calcutta. The government officials had also attended Congress sessions. With the increase in
Congress demands, the government became unfriendly. It encouraged the Muslims to stay
away from the Congress. The only demand of the Congress granted by the British was the
expansion of the legislative councils by the Indian Councils Act of 1892.

www.insightsactivelearn.com 8
Total Marks : 200
Test-4 (Subject)
( INSTA Prelims Test Series 2021 )

The moderate political activity involved constitutional agitation within the confines of law and
showed a slow but orderly political progress. The Moderates believed that the British basically
wanted to be just to the Indians but were not aware of the real conditions. Therefore, if public
opinion could be created in the country and public demands be presented to the government
through resolutions, petitions, meetings, etc., the authorities would concede these demands
gradually.

10. Consider the following statements regarding the Green Term Ahead Market (GTAM):
1. Transactions through GTAM will be bilateral in nature with clear identification of
corresponding buyers and sellers.
2. Energy scheduled through GTAM contract shall be considered as deemed Renewable Purchase
Obligation (RPO) compliance of the buyer.

Which of the statements given above is/are correct?


A. 1 only
B. 2 only
C. Both 1 and 2
D. Neither 1 nor 2

Correct Answer : C

Answer Justification :

Both the statements given above are correct.

Union Power Minister recently launched pan-India Green Term Ahead Market
(GTAM) in electricity through video conferencing.

Benefits of Green Term Ahead Market (GTAM):

The introduction of GTAM platform would lessen the burden on Renewable Energy -rich
States and incentivize them to develop Renewable Energy capacity beyond their own
RPO.
This would promote Renewable Energy merchant capacity addition and help in achieving
Renewable Energy capacity addition targets of the country.
GTAM platform will lead to increase in number of participants in renewable energy
sector.
It will benefit buyers of Renewable Energy through competitive prices and transparent
and flexible procurement.
It will also benefit Renewable Energy sellers by providing access to pan- India market.
Green Term Ahead Market contracts will allow additional avenues to the Renewable
Energy generators for sale of renewable energy;
It enable obligated entities to procure renewable power at competitive prices to meet
their Renewable Purchase Obligations (RPO);
It provides a platform to environmentally conscious open access consumers and utilities

www.insightsactivelearn.com 9
Total Marks : 200
Test-4 (Subject)
( INSTA Prelims Test Series 2021 )

to buy green power.

Key features of GTAM:

1. Transactions through GTAM will be bilateral in nature with clear identification of


corresponding buyers and sellers, there will not be any difficulty in accounting for
RPO.

2. GTAM contracts will be segregated into Solar RPO & Non-Solar RPO as RPO targets are
also segregated.

3. Price discovery will take place on a continuous basis i.e. price time priority basis.
Subsequently, looking at the market conditions open auction can be introduced for daily &
weekly contracts.

4. Energy scheduled through GTAM contract shall be considered as deemed RPO


compliance of the buyer.

https://pib.gov.in/PressReleasePage.aspx?PRID=1650388

11. Consider the following pairs regarding the Salt


Satyagraha
Region Led by
1. Malabar C. Rajagopalachari
2. Madras Region K. Kelappan,
3. Odisha Gopalbandhu Chaudhuri

Which of the pairs given above is/are matched correctly?


A. 1 and 2 only
B. 2 only
C. 1, 2 and 3
D. 3 only

Correct Answer : D

www.insightsactivelearn.com 10
Total Marks : 200
Test-4 (Subject)
( INSTA Prelims Test Series 2021 )

Answer Justification :

Salt Satyagraha at Different Places

A brief survey of the nature of Civil Disobedience Movement in different parts of the
subcontinent is given below.

Tamil Nadu: In April 1930, C. Rajagopalachari organised a march from


Thiruchirapalli (Trichinapoly as it was called by the British) to Vedaranniyam on the
Tanjore (or Thanjavur) coast to break the salt law.

The event was followed by widespread picketing of foreign cloth shops; the anti-liquor
campaign gathered forceful support in interior regions of Coimbatore, Madura, Virdhanagar,
etc. Although, Rajaji tried to keep the movement non-violent, violent eruptions of masses and
the violent repressions of the police
began. To break the Choolai mills strike, police force was used. Unemployed weavers attacked
liquor shops and police pickets at Gudiyattam, while the peasants, suffering from falling
prices, rioted at Bodinayakanur in Madura.

Malabar: K. Kelappan, a Nair Congress leader famed for the Vaikom Satyagraha,
organised salt marches. P. Krishna Pillai, the future founder of the Kerala Communist
movement, heroically defended the national flag in the face of police lathi-charge on Calicut
beach in November 1930.

Orissa: Under Gopalbandhu Chaudhuri, a Gandhian leader, salt satyagraha proved


effective in the coastal regions of Balasore, Cuttack and Puri districts.

12. Which of the following events took place under Lord Curzon?
1. Appointment of Universities Commission
2. Police Commission under Sir Andrew Frazer
3. Ancient Monuments Preservation Act
4. Calcutta Corporation Act

Select the correct answer using the code given below:


A. 1, 2 and 3 only
B. 2, 3 and 4 only
C. 1, 2 and 4 only
D. 1, 2, 3 and 4

Correct Answer : D

www.insightsactivelearn.com 11
Total Marks : 200
Test-4 (Subject)
( INSTA Prelims Test Series 2021 )

Answer Justification :

All the events given above took place Lord Curzon.

Lord Curzon (1899-1905)

Lord Curzon occupies a high place among the rulers of British India like Lord Wellesley and
Lord Dalhousie. He was a thorough imperialist. In order to make the administration efficient,
Lord Curzon overhauled the entire administrative machinery. His internal administration may
be studied under the following heads.

Educational Reforms

Curzon took a serious view of the fall in the standard of education and discipline in the
educational institutions. In his view the universities had degenerated into factories for
producing political revolutionaries. To set the educational system in order, he instituted in
1902, a Universities Commission to go into the entire question of university education in
the country.

Police and Military Reforms

Curzon believed in efficiency and discipline. He instituted a Police Commission in 1902 under
the chairmanship of Sir Andrew Frazer.

Calcutta Corporation Act (1899)

The Viceroy brought in a new legislative measure namely the Calcutta Corporation Act in 1899
by which the strength of the elected members was reduced and that of the official members
increased.

Preservation of Archaeological objects

Curzon had a passion for preserving the ancient monuments of historical importance in India.
No Viceroy in India before or after him took such a keen interest in archaeological objects. He
passed a law called the Ancient Monuments Act, 1904 which made it obligatory on the part of
the government and local authorities to preserve the monuments of archaeological importance
and their destruction an offence.

Lord Curzon 1899-1905

www.insightsactivelearn.com 12
Total Marks : 200
Test-4 (Subject)
( INSTA Prelims Test Series 2021 )

(i) Appointment of Police Commission (1902) under Sir Andrew Frazer to review police
administration.

(ii) Appointment of Universities Commission (1902) and passing of Indian Universities Act
(1904).

(iii) Establishment of Department of Commerce and Industry.

(iv) Calcutta Corporation Act (1899).

(v) Ancient Monuments Preservation Act (1904).

(vi) Partition of Bengal (1905).

(vii) Curzon-Kitchener controversy.

(viii) Younghusband’s Mission to Tibet (1904).

13. The Bandung Conference, led to the establishment of

A. The South Asian Association for Regional Cooperation (SAARC)


B. The Mekong Ganga Cooperation (MGC)
C. The Non-Aligned Movement (NAM)
D. The East Asia Summit (EAS)

Correct Answer : C

Answer Justification :

In March 1947, Nehru organised the Asian Relations Conference, attended by more than
twenty countries. The theme of the conference was Asian independence and assertion on the
world stage.

The de-colonization initiative was carried forward further at the Asian leaders’
conference in Colombo in 1954, culminating in the AfroAsian Conference in Bandung,
Indonesia, in 1955. The Bandung Conference set the stage for the meeting of nations
at Belgrade and the birth of the Non-Aligned Movement.

Hence, option (c) is correct.

14. Consider the following statements regarding Khan Abdul Gaffar Khan
1. He had started the first Pushto political monthly Pukhtoon.
2. He had organised a volunteer brigade ‘Khudai Khidmatgars’, popularly known as the ‘Red-
Shirts’,

www.insightsactivelearn.com 13
Total Marks : 200
Test-4 (Subject)
( INSTA Prelims Test Series 2021 )

3. Gaffar Khan was also called Badshah Khan and Frontier Gandhi.

Which of the statements given above is/are correct?


A. 1 and 3 only
B. 2 only
C. 3 only
D. 1, 2 and 3

Correct Answer : D

Answer Justification :

All the statements given above are correct

Khan Abdul Gaffar Khan’s educational and social reform work among the Pathans had
politicized them. Gaffar Khan, also called Badshah Khan and Frontier Gandhi, had started the
first Pushto political monthly Pukhtoon and had organised a volunteer brigade ‘Khudai
Khidmatgars’, popularly known as the ‘Red-Shirts’, who were pledged to the freedom struggle
and non-violence.

15. Consider the following statements regarding the Syngas:


1. It is a fuel gas mixture consisting primarily of nitrogen and carbon dioxide
2. It is produced by the process of coal gasification.

Which of the statements given above is/are correct?


A. 1 only
B. 2 only
C. Both 1 and 2
D. Neither 1 nor 2

Correct Answer : B

Answer Justification :

Coal gasification is the process of producing syngas—a mixture consisting primarily


of carbon monoxide (CO), hydrogen (H2), carbon dioxide (CO2), natural gas
consisting of mainly Methane (CH4), and water vapour (H2O)—from coal and water,
air and/or oxygen.

Hence, statement 1 is incorrect and statement 2 is correct.

Syngas can be used to produce a wide range of fertilizers, fuels, solvent and synthetic
materials.

www.insightsactivelearn.com 14
Total Marks : 200
Test-4 (Subject)
( INSTA Prelims Test Series 2021 )

Indian government has provided for a concession of 20% on revenue share of coal used
for gasification

http://ddnews.gov.in/business/india-aims-achieve-100-mt-coal-gasification-target-2030-union-m
inister-pralhad-joshi.

16. Consider the following statements regarding Indian National Congress


1. A British committee of the Indian National Congress was established in London in 1889.
2. A session of the Indian National Congress was held in London in 1892.

Which of the statements given above is/are correct?


A. 1 only
B. 2 only
C. Both 1 and 2
D. Neither 1 nor 2

Correct Answer : A

Answer Justification :

A British committee of the Indian National Congress was established in London in 1889 which
had India as its organ. Dadabhai Naoroji spent a substantial portion of his life and income
campaigning for India’s case abroad. In 1890, it was decided to hold a session of the
Indian National Congress in London in 1892, but owing to the British elections of
1891 the proposal was postponed and never revived later. Hence, statement 1 is
correct and statement 2 is incorrect.

17. Which of the following leaders led Dharasana Salt Satyagraha?

A. Abbas Tayabji, Surya Sen and Kasturbhai Lakhai


B. K.M. Ashraf Mohammed, Yasin Khan and Satyamurthy
C. N.V. Gadgil, Jadunandan Sharma and Tarunaram Phookan
D. Sarojini Naidu, Imam Sahib and Manilal

Correct Answer : D

Answer Justification :

www.insightsactivelearn.com 15
Total Marks : 200
Test-4 (Subject)
( INSTA Prelims Test Series 2021 )

On May 21, 1930, Sarojini Naidu, Imam Sahib and Manilal (Gandhi’s son) took up the
unfinished task of leading a raid on the Dharasana Salt Works. The unarmed and
peaceful crowd was met with a brutal lathicharge which left 2 dead and 320 injured. This new
form of salt satyagraha was eagerly adopted by people in Wadala (Bombay), Karnataka
(Sanikatta Salt Works), Andhra, Midnapore, Balasore, Puri and Cuttack.

18. What was the main objective of the Operation Barga?

A. Abolition of the Privy Purse


B. Abolition of Zamindari system in Karnataka
C. Police action against Nizam of Hyderabad
D. None of the above are correct.

Correct Answer : D

Answer Justification :

In West Bengal, the programme to confer tenancy rights was called Operation Barga.

This was quite successful, but the Communist government was criticized severely for
giving official sanction to tenancy (as opposed to giving tenant farmers ownership
rights to the land).

Hence, option (d) is correct

19. Consider the following statements regarding the Ilbert Bill controversy
1. The Ilbert Bill was presented in the Central Legislature under Lord Ripon.
2. The Bill tried to remove racial inequality between Indian and European judges in courts.

Which of the statements given above is/are correct?


A. 1 only
B. 2 only
C. Both 1 and 2
D. Neither 1 nor 2

Correct Answer : C

Answer Justification :

Both the statements given above are correct.

The Ilbert Bill controversy

www.insightsactivelearn.com 16
Total Marks : 200
Test-4 (Subject)
( INSTA Prelims Test Series 2021 )

The Ilbert Bill was presented in the Central Legislature during the Viceroyalty of Lord
Ripon. The Bill tried to remove racial inequality between Indian and European judges
in courts. This Bill was opposed by the British residents in India. Ultimately the Bill was
modified.

Ripon’s Government had sought to abolish “judicial disqualification based on race distinctions”
and to give the Indian members of the covenanted civil service the same powers and rights as
those enjoyed by their European colleagues. Ripon had to modify the bill, thus almost
defeating the original purpose, because of the stiff opposition from the European community.
It became clear to the nationalists that justice and fair play could not be expected where
interests of the European community were involved. However, the organised agitation by the
Europeans to revoke the Ilbert Bill also taught the nationalists how to agitate for certain rights
and demands.

20. Consider the following statements regarding the "Pradhan Mantri Kisan Sampada Yojana (PMKSY):
1. It is a central sector scheme.
2. The objective of the scheme is to supplement agriculture, modernize processing and decrease
Agri-Waste.
3. Mega Food Parks scheme is implemented under PMKSY.

Which of the statements given above is/are correct?


A. 1 and 2 only
B. 2 and 3 only
C. 1 and 3 only
D. 1, 2 and 3

Correct Answer : D

Answer Justification :

All the statements given above are correct.

The Central Sector Scheme - SAMPADA (Scheme for Agro-Marine Processing and
Development of Agro-Processing Clusters) was approved by the cabinet in May 2017 for
the period of 2016-20 coterminous with the 14th Finance Commission cycle. The scheme has
now been renamed as the "Pradhan Mantri Kisan Sampada Yojana (PMKSY)".

It is an umbrella scheme incorporating ongoing schemes of the Ministry like Mega


Food Parks, Integrated Cold Chain and Value Addition Infrastructure, Food Safety
and Quality Assurance Infrastructure, etc. and also new schemes like Infrastructure for
Agro-processing Clusters, Creation of Backward and Forward Linkages, Creation / Expansion
of Food Processing & Preservation Capacities.

www.insightsactivelearn.com 17
Total Marks : 200
Test-4 (Subject)
( INSTA Prelims Test Series 2021 )

The objective of PMKSY is to supplement agriculture, modernize processing and


decrease Agri-Waste.

Impact:

The implementation of PMKSY will result in creation of modern infrastructure with


efficient supply chain management from farm gate to retail outlet.
It will provide a big boost to the growth of food processing sector in the country.
It will help in providing better prices to farmers and is a big step towards doubling of
farmers’ income.
It will create huge employment opportunities especially in the rural areas.
It will also help in reducing wastage of agricultural produce, increasing the processing
level, availability of safe and convenient processed foods at affordable price to
consumers and enhancing the export of the processed foods.

https://www.businesstoday.in/current/economy-politics/govt-nod-to-27-cold-chains-in-11-states-
with-grant-in-aid-worth-rs-208-crore/story/414883.html

https://vikaspedia.in/agriculture/policies-and-schemes/crops-related/pradhan-mantri-kisan-sam
pada-yojana

21. Consider the following pairs of foundational theories of INC and their prominent believers
1. Safety Valve Theory —Lala Lajpat Rai
2. Conspiracy Theory—R.P. Dutt
3. Lightning conductor Theory—G.K. Gokhale

Which of the pairs given above is/are correctly matched?


A. 1 only
B. 1 and 3 only
C. 2 and 3 only
D. 1, 2 and 3

Correct Answer : D

Answer Justification :

All the pairs given above are correctly matched.

Foundational theories of INC and prominent believers:

Safety Valve Theory —Lala Lajpat Rai

Conspiracy Theory—R.P. Dutt

Lightning conductor Theory—G.K. Gokhale

www.insightsactivelearn.com 18
Total Marks : 200
Test-4 (Subject)
( INSTA Prelims Test Series 2021 )

There is a theory that Hume formed the Congress with the idea that it would prove to be a
‘safety valve’ for releasing the growing discontent of the Indians. To this end, he convinced
Lord Dufferin not to obstruct the formation of the Congress. The extremist leaders like Lala
Lajpat Rai believed in the ‘safety valve’ theory. Even the Marxist historian’s ‘conspiracy
theory’ was an offspring of the ‘safety valve’ notion. For example, R.P. Dutt opined that the
Indian National Congress was born out of a conspiracy to abort a popular uprising in India and
the bourgeois leaders were a party to it.

In the circumstances, as Bipan Chandra observes, the early Congress leaders used Hume as a
‘lightning conductor’ i.e., as a catalyst to bring together the nationalistic forces even if under
the guise of a ‘safety valve’.

22. Who was the Viceroy of India during the onset of Salt Satyagraha?

A. Lord Irwin
B. Lord Reading
C. Lord Hardinge II
D. Lord Willingdon

Correct Answer : A

Answer Justification :

Lord Irwin (1926 – 1931)

Important Events

Launch of civil disobedience movement and Dandi march


First round table conference was held
Lahore session of the Congress (1929); Purna Swaraj Resolution.
An All-Parties Conference held at Lucknow (1928) for suggestions for the (future)
Constitution of India, the report of which was called the Nehru Report or the Nehru
Constitution.
Appointment of the Harcourt Butler Indian States Commission (1927)

23. Which of the following is/are the provisions of the Trade Union Act, 1926?
1. It recognized trade unions as legal associations.
2. It laid down conditions for registration and regulation of trade union activities.
3. It secured immunity, both civil and criminal, for trade unions from prosecution for legitimate
activities.

www.insightsactivelearn.com 19
Total Marks : 200
Test-4 (Subject)
( INSTA Prelims Test Series 2021 )

Which of the statements given above is/are correct?


A. 1 and 2 only
B. 1 and 3 only
C. 2 and 3 only
D. 1, 2 and 3

Correct Answer : D

Answer Justification :

All the statements given above are correct.

Trade Union Act, 1926:

An Act to provide for the registration of Trade Unions and in certain respects to
define the law relating to registered Trade Unions.

The Trade Union Act, 1926

Recognized trade unions as legal associations;


Laid down conditions for registration and regulation of trade union activities;
Secured immunity, both civil and criminal, for trade unions from prosecution for
legitimate activities, but put some restrictions on their political activities.

24. Which of the following is/are basic tenets of Militant School of Thought?
1. Hatred for foreign rule
2. Swaraj to be the goal of national movement
3. Direct political action required
4. Belief in capacity of the masses to challenge the authority

Select the correct answer using the code given below:


A. 1, 2 and 3 only
B. 2, 3 and 4 only
C. 1, 2 and 4 only
D. 1, 2, 3 and 4

Correct Answer : D

Answer Justification :

All the statements given above are correct.

www.insightsactivelearn.com 20
Total Marks : 200
Test-4 (Subject)
( INSTA Prelims Test Series 2021 )

Militant School of Thought

By the dawn of the twentieth century, a band of nationalist thinkers had emerged who
advocated a more militant approach to political work. These included Raj Narain Bose, Ashwini
Kumar Datta, Aurobindo Ghosh and Bipin Chandra Pal in Bengal; Vishnu Shastri Chiplunkar
and Bal Gangadhar Tilak in Maharashtra; and Lala Lajpat Rai in Punjab. Tilak emerged as the
most outstanding representative of this school of thought.

The basic tenets of this school of thought were:

● hatred for foreign rule; since no hope could be derived from it, Indians should work
out their own salvation;

● swaraj to be the goal of national movement;

● direct political action required;

● belief in capacity of the masses to challenge the authority;

● personal sacrifices required and a true nationalist to be always ready for it.

25. The National Company Law Appellate Tribunal (NCLAT) hears appeals against the orders of which of the
following authorities/bodies?
1. Insolvency and Bankruptcy Board of India
2. National Company Law Tribunal(s) (NCLT)
3. Competition Commission of India (CCI)

Select the correct answer using the code given below:


A. 1 and 2 only
B. 1 and 3 only
C. 2 and 3 only
D. 1, 2 and 3

Correct Answer : D

Answer Justification :

The National Company Law Appellate Tribunal (NCLAT) hears appeals against the
orders of all the above authorities.

National Company Law Appellate Tribunal (NCLAT) was constituted under Section
410 of the Companies Act, 2013 for hearing appeals against the orders of National
Company Law Tribunal(s) (NCLT), with effect from 1st June, 2016.

www.insightsactivelearn.com 21
Total Marks : 200
Test-4 (Subject)
( INSTA Prelims Test Series 2021 )

NCLAT is also the Appellate Tribunal for hearing appeals against the orders passed by
NCLT(s) under Section 61 of the Insolvency and Bankruptcy Code, 2016 (IBC), with effect from
1st December, 2016. NCLAT is also the Appellate Tribunal for hearing appeals against
the orders passed by Insolvency and Bankruptcy Board of India under Section 202
and Section 211 of IBC.

NCLAT is also the Appellate Tribunal to hear and dispose of appeals against any
direction issued or decision made or order passed by the Competition Commission of
India (CCI) – as per the amendment brought to Section 410 of the Companies Act, 2013 by
Section 172 of the Finance Act, 2017, with effect from 26th May, 2017.

The decisions of the NCLAT can be appealed to the Supreme Court on a point of law.

https://www.thehindu.com/news/national/supreme-court-directs-telcos-to-pay-agr-dues-in-10-ye
ars/article32493787.ece

https://nclat.nic.in/?page_id=113

26. Consider the following statements regarding Swadeshi Movement:


1. In 1905, the Boycott Resolution was passed at Mumbai’s historic Gowalia Tank Maidan and the
formal proclamation of Swadeshi Movement was made.
2. Hitabadi, Sanjibani and Bengalee were used to propagate the agenda of Swadeshi Movement.

Which of the statements given above is/are correct?


A. 1 only
B. 2 only
C. Both 1 and 2
D. Neither 1 nor 2

Correct Answer : B

Answer Justification :

As a precursor to India’s independence, the Quit India Movement was launched at


Mumbai’s historic Gowalia Tank Maidan, now popularly known as August Kranti
Maidan on 8th August 1942. Hence, statement 1 is incorrect.

Mahatma Gandhi’s clarion call of ‘Do or Die’ inspired thousands of party workers but also
created a frenzy among the British who rushed to imprison the entire Congress leadership.

www.insightsactivelearn.com 22
Total Marks : 200
Test-4 (Subject)
( INSTA Prelims Test Series 2021 )

Anti-Partition Campaign Under Moderates (1903-05)

In the period 1903-1905, the leadership was provided by men like Surendranath Banerjea, K.K.
Mitra and Prithwish chandra Ray. The methods adopted were petitions to the
government, public meetings, memoranda, and propaganda through pamphlets and
newspapers such as Hitabadi, Sanjibani and Bengalee. Hence, statement 2 is correct.

Their objective was to exert sufficient pressure on the government through an educated public
opinion in India and England to prevent the unjust partition of Bengal from being
implemented.

Ignoring a loud public opinion against the partition proposal, the government announced
partition of Bengal in July 1905. Within days, protest meetings were held in small towns all
over Bengal. It was in these meetings that the pledge to boycott foreign goods was first taken.
On August 7, 1905, with the passage of the Boycott Resolution in a massive meeting
held in the Calcutta Townhall, the formal proclamation of Swadeshi Movement was
made. After this, the leaders dispersed to other parts of Bengal to propagate the message of
boycott of Manchester cloth and Liverpool salt.

27. Consider the following statements regarding Gandhi-Irwin Pact


1. Gandhi agreed to participate in the next Round Table Conference.
2. Irwin on behalf of the government agreed on an immediate release of all political prisoners not
convicted of violence.

Which of the statements given above is/are correct?


A. 1 only
B. 2 only
C. Both 1 and 2
D. Neither 1 nor 2

Correct Answer : C

Answer Justification :

Both the statements are correct.

Gandhi-Irwin Pact

On January 25, 1931, Gandhi and all other members of the Congress Working
Committee (CWC) were released unconditionally.

Gandhi agreed to participate in the next Round Table Conference.

The CWC authorised Gandhi to initiate discussions with the viceroy. As a result of these
discussions, a pact was signed between the viceroy, representing the British Indian

www.insightsactivelearn.com 23
Total Marks : 200
Test-4 (Subject)
( INSTA Prelims Test Series 2021 )

Government, and Gandhi, representing the Indian people, in Delhi on February 14, 1931.

This Delhi Pact, also known as the Gandhi-Irwin Pact, placed the Congress on an equal footing
with the government. Irwin on behalf of the government agreed on—

1. immediate release of all political prisoners not convicted of violence;


2. remission of all fines not yet collected;
3. return of all lands not yet sold to third parties;
4. lenient treatment to those government servants who had resigned;

5. right to make salt in coastal villages for personal consumption (not for sale);
6. right to peaceful and non-aggressive picketing; and
7. withdrawal of emergency ordinances.

28. The word swaraj was mentioned for the first time in which of the following congress session?

A. Congress session held at Lahore (1929)


B. Congress session held at Banaras (1905)
C. Congress session held at Calcutta (1906)
D. Congress session held at Surat (1907)

Correct Answer : C

Answer Justification :

The Indian National Congress, meeting in 1905 under the presidentship of Gokhale,
resolved to (i) condemn the partition of Bengal and the reactionary policies of
Curzon, and (ii) support the anti-partition and Swadeshi Movement of Bengal.

The militant nationalists led by Tilak, Lajpat Rai, Bipin Chandra Pal and Aurobindo Ghosh
wanted the movement to be taken outside Bengal to other parts of the country and go beyond
a boycott of foreign goods to become a fullfledged political mass struggle with the goal of
attaining swaraj. But the Moderates, dominating the Congress at that time, were not willing to
go that far.

However, a big step forward was taken at the Congress session held at Calcutta
(1906) under the presidentship of Dadabhai Naoroji, where it was declared that the
goal of the Indian National Congress was “self-government or swaraj like the United
Kingdom or the colonies” of Australia or Canada.

Hence, option (c) is correct.

The word swaraj was mentioned for the first time, but its connotation was not spelt
out, which left the field open for differing interpretations by the Moderates and the
Extremists.

www.insightsactivelearn.com 24
Total Marks : 200
Test-4 (Subject)
( INSTA Prelims Test Series 2021 )

The Moderate-Extremist dispute over the pace of the movement and techniques of struggle
reached a deadlock at the Surat session of the Indian National Congress (1907) where the
party split with serious consequences for the Swadeshi Movement.

29. Consider the following statements:


1. Sorabjee Shapoorji Bengalee started a workingmen’s club.
2. Narain Meghajee Lokhanday started the newspaper Bharat Shramjeevi
3. Sasipada Banerjea set up the Bombay Mill and Millhands Association.

Which of the statements given above is/are correct?


A. 1 only
B. 2 and 3 only
C. 1, 2 and 3
D. None

Correct Answer : D

Answer Justification :

None of the statements given above are correct.

Earlier attempts to improve the conditions of the workers:

Sasipada Banerjea started a workingmen’s club and newspaper Bharat


Shramjeevi.
Sorabjee Shapoorji Bengalee tried to get a bill, providing better working
conditions to labour, passed in the Bombay Legislative Council.
Narain Meghajee Lokhanday started the newspaper Deenbandhu and set up the
Bombay Mill and Millhands Association.

30. Consider the following statements regarding the National Food Security Act, 2013:
1. The Act provides for coverage of upto 50% of the rural population and upto 75% of the urban
population for receiving subsidized food grains under Targeted Public Distribution System
(TPDS).
2. The Act provides that every State Government shall, by notification, constitute a State Food
Commission for the purpose of monitoring and review of implementation of the Act.
3. The work of identification of eligible households is to be done by States/UTs.

Which of the statements given above is/are correct?

www.insightsactivelearn.com 25
Total Marks : 200
Test-4 (Subject)
( INSTA Prelims Test Series 2021 )

A. 1 and 2 only
B. 2 and 3 only
C. 1 and 3 only
D. 1, 2 and 3

Correct Answer : B

Answer Justification :

The National Food Security Act, 2013 was notified on 10th September, 2013 with the objective
to provide for food and nutritional security in human life cycle approach, by ensuring access to
adequate quantity of quality food at affordable prices to people to live a life with dignity.

The Act provides for coverage of upto 75% of the rural population and upto 50% of
the urban population for receiving subsidized foodgrains under Targeted Public
Distribution System (TPDS), thus covering about two-thirds of the population.

Hence, statement 1 is incorrect.

The eligible persons will be entitled to receive 5 Kgs of foodgrains per person per month at
subsidised prices of Rs. 3/2/1 per Kg for rice/wheat/coarse grains. The existing Antyodaya
Anna Yojana (AAY) households, which constitute the poorest of the poor, will continue to
receive 35 Kgs of food grains per household per month.

Corresponding to the all India coverage of 75% and 50% in the rural and urban areas,
State-wise coverage under NFSA was determined by the erstwhile Planning
Commission (now NITI Aayog) by using the NSS Household Consumption Survey data
for 2011-12. Within the coverage under TPDS determined for each State, the work of
identification of eligible households is to be done by States/UTs. It is the
responsibility of the State Governments/UTs, to evolve criteria for identification of
priority households and their actual identification.

Hence, statement 3 is correct.

Section 16 of the National Food Security Act, 2013 (NFSA provides that every State
Government shall, by notification, constitute a State Food Commission for the
purpose of monitoring and review of implementation of the Act. The State Food
Commission shall consist of a Chairperson, five other Members and a Member-Secretary with
associated administrative and technical staff. (2 women and SC/ST).

Hence, statement 2 is correct.

https://www.thehindu.com/news/national/states-asked-to-identify-pwds-for-ration-cards/article
32474070.ece

www.insightsactivelearn.com 26
Total Marks : 200
Test-4 (Subject)
( INSTA Prelims Test Series 2021 )

https://dfpd.gov.in/nfsa-act.htm

31. Consider the following statements regarding Delhi Proposals of Muslim League
1. Joint electorates in place of separate electorates with reserved seats for Muslims
2. Formation of three new Muslim majority provinces
3. One-third representation to Muslims in Central Legislative Assembly

Which of the statements given above is/are correct?


A. 1 only
B. 1 and 3 only
C. 2 and 3 only
D. 1, 2 and 3

Correct Answer : D

Answer Justification :

All the statements given above are correct.

Delhi Proposals of Muslim League

Earlier, in December 1927, a large number of Muslim leaders had met at Delhi at the Muslim
League session and evolved four proposals for their demands to be incorporated into the draft
constitution. These proposals, which were accepted by the Madras session of the Congress
(December 1927), came to be known as the ‘Delhi Proposals’. These were:

● joint electorates in place of separate electorates with reserved seats for Muslims;

● one-third representation to Muslims in Central Legislative Assembly;

● representation to Muslims in Punjab and Bengal in proportion to their population;

● formation of three new Muslim majority provinces— Sindh, Baluchistan and North-
West Frontier Province.

32. Consider the following statements regarding Government of India Act of 1935
1. It provided for the establishment of an All-India Federation consisting of provinces and
princely states as units.
2. It abolished dyarchy in the provinces and introduced ‘provincial autonomy’ in its place.

www.insightsactivelearn.com 27
Total Marks : 200
Test-4 (Subject)
( INSTA Prelims Test Series 2021 )

Which of the statements given above is/are correct?


A. 1 only
B. 2 only
C. Both 1 and 2
D. Neither 1 nor 2

Correct Answer : C

Answer Justification :

Both the statements are correct.

Government of India Act of 1935

The Act marked a second milestone towards a completely responsible government in India. It
was a lengthy and detailed document having 321 Sections and 10 Schedules.

Features of the Act

1. It provided for the establishment of an All-India Federation consisting of provinces


and princely states as units. The Act divided the powers between the Centre and units in
terms of three lists—Federal List (for Centre, with 59 items), Provincial List (for provinces,
with 54 items) and the Concurrent List (for both, with 36 items). Residuary powers were given
to the Viceroy. However, the federation never came into being as the princely states did not
join it.

2. It abolished dyarchy in the provinces and introduced ‘provincial autonomy’ in its


place. The provinces were allowed to act as autonomous units of administration in their
defined spheres. Moreover, the Act introduced responsible governments in provinces, that is,
the governor was
required to act with the advice of ministers responsible to the provincial legislature. This came
into effect in 1937 and was discontinued in 1939.

3. It provided for the adoption of dyarchy at the Centre. Consequently, the federal subjects
were divided into reserved subjects and transferred subjects. However, this provision of the
Act did not come into operation at all.

4. It introduced bicameralism in six out of eleven provinces. Thus, the legislatures of Bengal,
Bombay, Madras, Bihar, Assam and the United Provinces were made bicameral consisting of a
legislative council (upper house) and a legislative assembly (lower house). However, many
restrictions were placed on them.
5. It further extended the principle of communal representation by providing separate
electorates for depressed classes (scheduled castes), women and labour (workers).

www.insightsactivelearn.com 28
Total Marks : 200
Test-4 (Subject)
( INSTA Prelims Test Series 2021 )

33. Consider the following pairs of newspapers and


their Founder/Editor:
Newspaper: Founder/Editor
1. Voice of India Dadabhai Naoroji
2. Sudharak Gopal Ganesh Agarkar
3. Swadesamitran G. Subramania Iyer

Which of the pairs given above is/are correctly matched?


A. 1 and 2 only
B. 2 and 3 only
C. 1 and 3 only
D. 1, 2 and 3

Correct Answer : D

Answer Justification :

All the pairs given above are correctly matched.

Dadabhai Naoroji founded the East India Association in London, one of the predecessor
organisations of the Indian National Congress with the aim of putting across the Indian point
of view before the British public. In 1874 he was appointed the Dewan of Baroda and a year
later, on account of differences with the Maharaja and the Resident, he resigned from the
Dewanship. In July 1875 he was elected a Member of the Municipal Corporation, Bombay. In
1876 he resigned and left for London. He was appointed as Justice of the Peace in 1883,
started a newspaper called 'Voice of India' and was elected to the Bombay Municipal
Corporation for the second time. In August 1885 he joined the Bombay Legislative Council at
the invitation of the Governor, Lord Reay.

Sudharak, meaning Reformer, was a newspaper in India. It was founded in 1888 by


Gopal Ganesh Agarkar, who had previously edited Kesari. The Newspaper was an Anglo-
Marathi-language work and was published in the city of Pune in the present day Indian state of
Maharashtra.

Swadesamitran was a Tamil language newspaper that was published from the then
Madras city from 1882 to 1985. One of the earliest Tamil newspapers and the longest
in print, Swadesamitran was founded by Indian nationalist G. Subramania Iyer four
years after he had started The Hindu.

www.insightsactivelearn.com 29
Total Marks : 200
Test-4 (Subject)
( INSTA Prelims Test Series 2021 )

34. Consider the following statements regarding the promotion of National Education
1. The National Council of Education was set up in 1906
2. Bengal National College was inspired by Tagore’s Shantiniketan

Which of the statements given above is/are correct?


A. 1 only
B. 2 only
C. Both 1 and 2
D. Neither 1 nor 2

Correct Answer : C

Answer Justification :

All the statements given above are correct.

Programme of Swadeshi or National Education

Bengal National College, inspired by Tagore’s Shantiniketan, was set up with Aurobindo
Ghosh as its principal. Soon national schools and colleges sprang up in various parts of the
country. On August 15, 1906, the National Council of Education was set up to organise
a system of education— literary, scientific and technical—on national lines and under
national control. Education was to be imparted through the vernacular medium. A Bengal
Institute of Technology was set up for technical education and funds were raised to send
students to Japan for advanced learning.

35. Consider the following statements regarding the Wainganga River:


1. It rises in the Amarakantak hills of Madhya Pradesh.
2. It is a tributary of River Krishna.
3. The Wardha River is one of the major tributaries of Wainganga River.

Which of the statements given above is/are not correct?


A. 1 and 2 only
B. 2 and 3 only
C. 3 only
D. None

Correct Answer : A

Answer Justification :

The Wainganga is a river in India originating in the Mahadeo Hills in Mundara near the
village Gopalganj in Seoni, Madhya Pradesh.

www.insightsactivelearn.com 30
Total Marks : 200
Test-4 (Subject)
( INSTA Prelims Test Series 2021 )

Hence, statement 1 is incorrect.

The Wainganga (Pranhita) is one of the northward tributary of the river Godavari in its
reaches.

Hence, statement 2 is incorrect.

On the right bank five tributaries viz. the Bawanthari, the Kanhan, the Andhari, the
Wardha and the peddavagu joins the Wainganga. On the left bank 5 tributaries namely
the Bagh, the Chulband, the Garhvi, the Kobragarhi and the Kathani drains in the main
channel.

Hence, statement 3 is correct.

https://www.indiatoday.in/india/story/new-bridge-in-mp-s-seoni-district-collapses-on-a
ugust-30-official-date-of-completion-1716799-2020-08-30

http://www.cwc.gov.in/mco/wainganga-basin

36. Consider the following statements regarding Simon Commission recommendations


1. It proposed the abolition of dyarchy.
2. The report accepted parliamentary responsibility at the center.
3. The governor-general was to have complete power to appoint the members of the cabinet.

Which of the statements given above is/are correct?


A. 1 and 2 only
B. 1 and 3 only
C. 2 and 3 only
D. 1, 2 and 3

Correct Answer : B

Answer Justification :

The Simon Commission Recommendations

The Simon Commission published a two-volume report in May 1930. It proposed the
abolition of dyarchy and the establishment of representative government in the
provinces which should be given autonomy. It said that the governor should have
discretionary power in relation to internal security and administrative powers to protect the
different communities. The number of members of provincial legislative council should be
increased. Hence, statement 1 is correct.

The report rejected parliamentary responsibility at the centre. Hence, statement 2 is

www.insightsactivelearn.com 31
Total Marks : 200
Test-4 (Subject)
( INSTA Prelims Test Series 2021 )

incorrect.

The governor-general was to have complete power to appoint the members of the
cabinet. Hence, statement 3 is correct.

And the Government of India would have complete control over the high court. It also
recommended that separate communal electorates be retained (and extended such electorates
to other communities) but only until tensions between Hindus and Muslims had died down.
There was to be no universal franchise.

It accepted the idea of federalism but not in the near future; it suggested that a
Consultative Council of Greater India should be established which should include
representatives of both the British provinces as well as princely states.
It suggested that the North-West Frontier Province and Baluchistan should get local
legislatures, and both NWFP and Baluchistan should have the right to be represented at
the centre.
It recommended that Sindh should be separated from Bombay, and Burma should be
separated from India because it was not a natural part of the Indian subcontinent.
It also suggested that the Indian army should be Indianized though British forces must
be retained. India got fully equipped.
But by the time the report came out, it was no longer relevant because several events
overtook the importance of its recommendations.

37. Which of the following personalities have participated in First Round Table Conference
1. Motilal Nehru
2. Muhammad Ali Jinnah
3. B.R. Ambedkar

Select the correct answer using the code given below


A. 1 and 3 only
B. 2 and 3 only
C. 1 and 2 only
D. 1, 2 and 3

Correct Answer : B

Answer Justification :

The first Round Table Conference was held in London between November 1930 and
January 1931. It was opened officially by King George V on November 12, 1930 and chaired
by Ramsay MacDonald.

The Muslim League sent Aga Khan III (leader of British Indian delegation), Maulana
Mohammad Ali Jauhar, Muhammad Shafi, Muhammad Ali Jinnah, Muhammad Zafarullah
Khan, A.K. Fazlul Huq, Hafiz Ghulam Hussain Hidayat Ullah, Dr.Shafa’at Ahmad Khan, Raja
Sher Muhammad Khan of Domeli and A.H. Ghuznavi.

www.insightsactivelearn.com 32
Total Marks : 200
Test-4 (Subject)
( INSTA Prelims Test Series 2021 )

The Hindu Mahasabha and its sympathisers were represented by B.S. Moonje, M.R. Jayakar
and Diwan Bahadur Raja Narendra Nath. The Sikhs were represented by Sardar Ujjal Singh
and Sardar Sampuran Singh.

For the Parsis, Phiroze Sethna, Cowasji Jehangir and Homi Mody attended. Begum Jahanara
Shahnawaz and Radhabai Subbarayan represented Women. The Liberals were represented by
J.N. Basu, Tej Bahadur Sapru, C.Y. Chintamani, V.S. Srinivasa Sastri and Chimanlal Harilal
Setalvad. The Depressed Classes were represented by B.R. Ambedkar and Rettamalai
Srinivasan. Hence Statement 3 is correct.

Motilal Nehru didn’t attain round table conference.

38. Consider the following statements regarding the regulation of press during the British India:
1. Censorship of Press Act, 1799 imposed almost wartime press restrictions including pre-
censorship.
2. Lord Ripon enacted the Vernacular Press Act (VPA) to effectively restrict the vernacular press.
3. Indian Press Act, 1910 empowered the local government to demand a security at registration
from the printer/publisher.

Which of the statements given above is/are correct?


A. 1 and 2 only
B. 2 and 3 only
C. 1 and 3 only
D. 1, 2 and 3

Correct Answer : C

Answer Justification :

Censorship of Press Act, 1799:

Lord Wellesley enacted this, anticipating French invasion of India. It imposed almost
wartime press restrictions including pre-censorship. These restrictions were relaxed
under Lord Hastings, who had progressive views, and in 1818, pre-censorship was dispensed
with.

Hence, statement 1 is correct.

The Vernacular Press Act (VPA) was designed to ‘better control’ the vernacular press
and effectively punish and repress seditious writing. It was enacted by Lord Lytton
and later it was repealed by Lord Ripon.

Hence, statement 2 is incorrect.

www.insightsactivelearn.com 33
Total Marks : 200
Test-4 (Subject)
( INSTA Prelims Test Series 2021 )

Indian Press Act, 1910: This Act revived the worst features of the VPA—local
government was empowered to demand a security at registration from the
printer/publisher and fortfeit/deregister if it was an offending newspaper, and the printer of
a newspaper was required to submit two copies of each issue to local government free of
charge.

Hence, statement 3 is correct.

39. The Surat split took place due to differences on which of the following resolutions?
1. Swadeshi
2. Boycott
3. National education

Select the correct answer using the code given below:


A. 1 and 2 only
B. 1 and 3 only
C. 2 and 3 only
D. 1, 2 and 3

Correct Answer : D

Answer Justification :

Split Takes Place

The Extremists wanted the 1907 session to be held in Nagpur (Central Provinces) with Tilak or
Lajpat Rai as the president along with a reiteration of the swadeshi, boycott and national
education resolutions. The Moderates wanted the session at Surat in order to exclude Tilak
from the presidency, since a leader from the host province could not be session president
(Surat being in Tilak’s home province of Bombay). Instead, they wanted Rashbehari Ghosh
as the president and sought to drop the resolutions on swadeshi, boycott and national
education. Both sides adopted rigid positions, leaving no room for compromise. The split
became inevitable, and the Congress was now dominated by the Moderates who lost no time in
reiterating Congress’ commitment to the goal of self-government within the British Empire
and to the use of constitutional methods only to achieve this goal.

Major Cause of Moderate-Extremist Split at Surat (1907)

Moderates wanted to restrict the Boycott Movement to Bengal and to a boycott of foreign
cloth and liquor.

Extremists wanted to take the movement to all parts of the country and include within its
ambit all forms of association with the government through a boycott of schools, colleges, law
courts, legislative councils, government service, municipalities, etc.

www.insightsactivelearn.com 34
Total Marks : 200
Test-4 (Subject)
( INSTA Prelims Test Series 2021 )

40. Onam is an annual harvest festival of:

A. Andhra Pradesh
B. Telangana
C. Kerala
D. Karnataka

Correct Answer : C

Answer Justification :

Onam is an annual harvest festival of Kerala.

Onam is celebrated at the beginning of the month of Chingam, the first month of the
solar Malayalam calendar (Kollavarsham). It falls in August or September each year.

According to a popular legend, the festival is celebrated to welcome King Mahabali,


whose spirit is said to visit Kerala at the time of Onam.

It is also celebrated as the festival of paddy harvest.

How is it celebrated?

During the festival, people get dressed in their traditional attire, prepare the Onam Sadhya
and take part in a variety of group activities such as the Pookolam (rangoli made with
fresh flowers), Vallam Kali (boat race), Pulikali (tiger dance), Kai Kottu Kali (Onam
dance), Kummattikali (mask dance) and various other activities.

https://www.hindustantimes.com/india-news/president-kovind-pm-modi-extend-onam-greetings
/story-Y5yd1jdGOLdEIzcmTDiI0I.html

https://www.insightsonindia.com/2020/08/31/onam/

41. Consider the following statements regarding Morley- Minto (or Minto-Morley) Reforms
1. One Indian was to be appointed to the viceroy’s executive council.
2. The elected members were to be directly elected by the people of the country.
3. Separate electorates for Muslims for election to the central council was established.

Which of the statements given above is/are correct?


A. 1 only
B. 1 and 3 only

www.insightsactivelearn.com 35
Total Marks : 200
Test-4 (Subject)
( INSTA Prelims Test Series 2021 )

C. 2 and 3 only
D. 1, 2 and 3

Correct Answer : B

Answer Justification :

The viceroy, Lord Minto, and the Secretary of State for India, John Morley, agreed that some
reforms were due so as to placate the Moderates as well as the Muslims. They worked out a
set of measures that came to be known as the Morley- Minto (or Minto-Morley) Reforms
that translated into the Indian Councils Act of 1909.

● The elective principle was recognized for the nonofficial membership of the councils in
India. Indians were allowed to participate in the election of various legislative councils, though
on the basis of class and community.

● For the first time, separate electorates for Muslims for election to the central
council was established—a most detrimental step for India. Hence, statement 3 is
correct.

● The number of elected members in the Imperial Legislative Council and the Provincial
Legislative Councils was increased. In the provincial councils, non-official majority was
introduced, but since some of these non-officials were nominated and not elected, the overall
non-elected majority remained.

● According to Sumit Sarkar, in the Imperial Legislative Council, of the total 69


members, 37 were to be the officials and of the 32 non-officials, 5 were to be
nominated. Of the 27 elected non-officials, 8 seats were reserved for the Muslims
under separate electorates (only Muslims could vote here for the Muslim candidates),
while 4 seats were reserved for the British capitalists, 2 for the landlords and 13 seats
came under general electorate.

● The elected members were to be indirectly elected. Hence, statement 2 is incorrect.


The local bodies were to elect an electoral college, which in turn would elect members of
provincial legislatures, who in turn would elect members of the central legislature.

● Besides separate electorates for the Muslims, representation in excess of the strength of
their population was accorded to the Muslims. Also, the income qualification for Muslim voters
was kept lower than that for Hindus.

● Powers of legislatures—both at the center and in provinces—were enlarged and the


legislatures could now pass resolutions (which may or may not be accepted), ask questions and
supplementaries, vote separate items in the budget though the budget as a whole could not be
voted upon.

● One Indian was to be appointed to the viceroy’s executive council (Satyendra Sinha
was the first Indian to be appointed in 1909).

www.insightsactivelearn.com 36
Total Marks : 200
Test-4 (Subject)
( INSTA Prelims Test Series 2021 )

Hence, statement 1 is correct.

42. Consider the following statements regarding working of Congress Ministries in Provinces
1. Laws giving emergency powers were repealed.
2. Newspapers were taken out of black lists.
3. Political prisoners and revolutionaries were released

Which of the statements given above is/are correct?


A. 1 and 2 only
B. 2 and 3 only
C. 3 only
D. 1, 2 and 3

Correct Answer : D

Answer Justification :

All the statements given above are correct.

Working of Congress Ministries

Civil Liberties

The Congress ministries did much to ease curbs on civil liberties:


● Laws giving emergency powers were repealed.
● Ban on illegal organisations, such as the Hindustan Seva Dal and Youth Leagues, and on
certain books and journals was lifted.
● Press restrictions were lifted.
● Newspapers were taken out of black lists.
● Confiscated arms and arms licences were restored.
● Police powers were curbed and the CID stopped shadowing politicians.
● Political prisoners and revolutionaries were released, and deportation and
internment orders were revoked.
● In Bombay lands confiscated by the government during the Civil Disobedience Movement
were restored.
● Pensions of officials associated with the Civil Disobedience Movement were restored.

43. Consider the following statements regarding the Indian Independence Act, 1947:
1. It provided, for each dominion, a governor-general, who was to be appointed by the British
King on the advice of the dominion cabinet
2. It empowered the Constituent Assemblies of the two dominions to repeal any act of the British
Parliament, including the Independence act itself.
3. It abolished the office of the secretary of state for India.

www.insightsactivelearn.com 37
Total Marks : 200
Test-4 (Subject)
( INSTA Prelims Test Series 2021 )

Which of the statements given above is/are correct?


A. 1 and 2 only
B. 2 and 3 only
C. 1 and 3 only
D. 1, 2 and 3

Correct Answer : D

Answer Justification :

All the statements given above are correct.

Features of the Indian Independence Act of 1947:

1. It ended the British rule in India and declared India as an independent and sovereign state
from August 15, 1947.

2. It provided for the partition of India and creation of two independent dominions of India and
Pakistan with the right to secede from the British Commonwealth.

3. It abolished the office of viceroy and provided, for each dominion, a governor-
general, who was to be appointed by the British King on the advice of the dominion
cabinet. His Majesty’s Government in Britain was to have no responsibility with respect to the
Government of India or Pakistan.

4. It empowered the Constituent Assemblies of the two dominions to frame and adopt
any constitution for their respective nations and to repeal any act of the British
Parliament, including the Independence act itself.

5. It empowered the Constituent Assemblies of both the dominions to legislate for their
respective territories till the new constitutions were drafted and enforced. No Act of the
British Parliament passed after August 15, 1947 was to extend to either of the new dominions
unless it was extended thereto by a law of the legislature of the dominion.

6. It abolished the office of the secretary of state for India and transferred his
functions to the secretary of state for Commonwealth Affairs.

7. It proclaimed the lapse of British paramountcy over the Indian princely states and treaty
relations with tribal areas from August 15, 1947.

8. It granted freedom to the Indian princely states either to join the Dominion of India or
Dominion of Pakistan or to remain independent.

9. It provided for the governance of each of the dominions and the provinces by the
Government of India Act of 1935, till the new Constitutions were framed. The dominions were
however authorised to make modifications in the Act.

10. It deprived the British Monarch of his right to veto bills or ask for reservation of certain

www.insightsactivelearn.com 38
Total Marks : 200
Test-4 (Subject)
( INSTA Prelims Test Series 2021 )

bills for his approval. But, this right was reserved for the Governor-General. The Governor-
General would have full power to assent to any bill in the name of His Majesty.

11. It designated the Governor-General of India and the provincial governors as constitutional
(nominal) heads of the states. They were made to act on the advice of the respective council of
ministers in all matters.

12. It dropped the title of Emperor of India from the royal titles of the king of England.

13. It discontinued the appointment to civil services and reservation of posts by the secretary
of state for India. The members of the civil services appointed before August 15, 1947 would
continue to enjoy all benefits that they were entitled to till that time.

44. Consider the following statements regarding revolutionary activities


1. The first of the revolutionary activities in Maharashtra was the organization of the Ramosi
Peasant Force by Vasudev Balwant Phadke.
2. Savarkar and his brother organized Anushilan Samiti.

Which of the statements given above is/are correct?


A. 1 only
B. 2 only
C. Both 1 and 2
D. Neither 1 nor 2

Correct Answer : A

Answer Justification :

The first of the revolutionary activities in Maharashtra was the organization of the
Ramosi Peasant Force by Vasudev Balwant Phadke in 1879, which aimed to rid the
country of the British by instigating an armed revolt by disrupting communication lines. It
hoped to raise funds for its activities through dacoities. It was suppressed prematurely.

Hence, statement 1 is correct.

During the 1890s, Tilak propagated a spirit of militant nationalism, including use of violence,
through Ganapati and Shivaji festivals and his journals Kesari and Maharatta. Two of his
disciples—the Chapekar brothers, Damodar and Balkrishna—murdered the Plague
Commissioner of Poona, Rand, and one Lt. Ayerst in 1897. Savarkar and his brother
organised Mitra Mela, a secret society, in 1899 which merged with Abhinav Bharat
(after Mazzinni’s ‘Young Italy’) in 1904.

Hence, statement 2 is incorrect.

Soon Nasik, Poona and Bombay emerged as centres of bomb manufacture. In 1909, A.M.T.

www.insightsactivelearn.com 39
Total Marks : 200
Test-4 (Subject)
( INSTA Prelims Test Series 2021 )

Jackson, the Collector of Nasik, who was also a well-known indologist, was killed by Anant
Lakshman Kanhere, a member of Abhinav Bharat.

45. Consider the following statements regarding the Association of Renewable Energy Agencies of state
(AREAS):
1. It is an initiative of NITI Aayog.
2. Prime Minister of India is its ex-officio President.

Which of the statements given above is/are correct?


A. 1 only
B. 2 only
C. Both 1 and 2
D. Neither 1 nor 2

Correct Answer : D

Answer Justification :

None of the statements given above are correct.

Ministry of New & Renewable Energy (MNRE) is the nodal agency at the central level for
promotion of grid-connected and off-grid renewable energy in the country. Ministry’s
programmes are implemented in close coordination with State Nodal Agencies (SNAs) for
renewable energy (RE). Over the period the SNAs have developed considerable knowledge and
experience in planning and implementation of RE programmes. In this background it is
important that SNAs interact and learn from each other’s experiences and also share their
best practices and knowledge regarding technologies and schemes/programmes.

MNRE took an initiative in this regard in consultation with SNAs, and Association of
Renewable Energy Agencies of States (abbreviated as “AREAS) has been formed and
registered as a society on 27 August 2014 under Society Registration Act 1860.

Secretary, MNRE is the ex-officio President of the Association. All SNAs to be the
member of the Association.

https://pib.gov.in/Pressreleaseshare.aspx?PRID=1649208

https://mnre.gov.in/areas

46. Consider the following statements regarding the Communal Award


1. The Communal Award was based on the findings of the Indian Franchise Committee.
2. The Simon Commission accepted the proposal of separate electorate for the depressed classes.

Which of the statements given above is/are correct?


www.insightsactivelearn.com 40
Total Marks : 200
Test-4 (Subject)
( INSTA Prelims Test Series 2021 )

A. 1 only
B. 2 only
C. Both 1 and 2
D. Neither 1 nor 2

Correct Answer : A

Answer Justification :

The Communal Award was announced by the British prime minister, Ramsay MacDonald, on
August 16, 1932. The Communal Award, based on the findings of the Indian Franchise
Committee (also called the Lothian Committee), established separate electorates and
reserved seats for minorities, including the depressed classes which were granted seventy-
eight reserved seats. Hence Statement 1 is correct.

Thus, this award accorded separate electorates for Muslims, Europeans, Sikhs, Indian
Christians, Anglo-Indians, depressed classes, and even to the Marathas for some seats in
Bombay. The award was perceived by the national leaders led by the Congress as another
manifestation of the British policy of divide and rule.

It should be noted here that Dr B.R. Ambedkar in the past, in his testimony to the Simon
Commission, had stressed that the depressed classes should be treated as a distinct,
independent minority separate from the caste Hindus. Even, the Bengal Depressed Classes
Association had lobbied for separate electorates with seats reserved according to the
proportion of depressed class members to the total population as well as for adult franchise.
But the Simon Commission rejected the proposal of separate electorate for the
depressed classes; however, it retained the concept of reserving seats. Hence
Statement 2 is incorrect.

47. The Satnami movement in central India which worked to improve the social status of the leatherworkers was
founded by:

A. Birsa Munda
B. Haridas Thakur
C. Ghasidas
D. Bakshi Jagabandhu

Correct Answer : C

Answer Justification :

The Satnami movement in Central India was founded by Ghasidas who worked among
the leatherworkers and organised a movement to improve their social status.

www.insightsactivelearn.com 41
Total Marks : 200
Test-4 (Subject)
( INSTA Prelims Test Series 2021 )

Hence, option (c) is correct.

48. Consider the following statements regarding Poona Pact


1. It largely achieved the desired goal of emancipation of the depressed class
2. The seats reserved for the depressed classes were increased from 71 to 147 in provincial
legislatures under Poona Pact

Which of the statements given above is/are correct?


A. 1 only
B. 2 only
C. Both 1 and 2
D. Neither 1 nor 2

Correct Answer : B

Answer Justification :

Signed by B.R. Ambedkar on behalf of the depressed classes on September 24, 1932, the
Poona Pact abandoned the idea of separate electorates for the depressed classes.

But the seats reserved for the depressed classes were increased from 71 to 147 in
provincial legislatures and to 18 per cent of the total in the Central Legislature.
Hence statement 2 is correct.

The Poona Pact was accepted by the government as an amendment to the Communal Award.
Impact of Poona Pact on Dalits

The Poona Pact, despite giving certain political rights to the depressed classes, could
not achieve the desired goal of emancipation of the depressed class. It enabled the
same old Hindu social order to continue and gave birth to many problems. Hence Statement
1 is incorrect.

The Pact made the depressed classes political tools which could be used by the majoritarian
caste Hindu organisations.

It made the depressed classes leaderless as the true representatives of the classes were
unable to win against the stooges who were chosen and supported by the caste Hindu
organisations.

This led to the depressed classes to submit to the status quo in political, ideological and
cultural fields and not being able to develop independent and genuine leadership to fight the
Brahminical order.

49. Consider the following statements regarding revolutionary activities in foreign land

www.insightsactivelearn.com 42
Total Marks : 200
Test-4 (Subject)
( INSTA Prelims Test Series 2021 )

1. Virendranath Chattopadhyay had started an Indian Home Rule Society in London in 1905.
2. The Berlin Committee for Indian Independence was established in 1915 by Shyamji
Krishnavarma.

Which of the statements given above is/are correct?


A. 1 only
B. 2 only
C. Both 1 and 2
D. Neither 1 nor 2

Correct Answer : D

Answer Justification :

Shyamji Krishnavarma had started in London in 1905 an Indian Home Rule


Society—‘India House’—as a centre for Indian students, a scholarship scheme to bring
radical youth from India, and a journal The Indian Sociologist. Hence, statement 1 is
incorrect.

Revolutionaries such as Savarkar and Hardayal became the members of India House. Madanlal
Dhingra from this circle assassinated the India office bureaucrat Curzon-Wyllie in 1909. Soon,
London became too dangerous for the revolutionaries, particularly after Savarkar had been
extradited in 1910 and transported for life in the Nasik conspiracy case.

New centres emerged on the continent—Paris and Geneva—from where Madam Bhikaji Cama,
a Parsi revolutionary who had developed contacts with Frenchsocialists and who brought out
Bande Mataram, and Ajit Singh operated. And after 1909 when Anglo-German relations
deteriorated, Virendranath Chattopadhyaya chose Berlin as his base.

The Berlin Committee for Indian Independence was established in 1915 by


Virendranath Chattopadhyay, Bhupendranath Dutta, Lala Hardayal and others with
the help of the German foreign office under ‘Zimmerman Plan’. Hence, statement 2 is
incorrect.

50. Consider the following statements regarding the Atal Pension Yojana (APY):
1. It encourages the workers in an unorganized sector to voluntarily save for their retirement.
2. Contributions to the Atal Pension Yojana (APY) is eligible for tax benefits similar to the
National Pension System (NPS).
3. The minimum age of joining APY is 21 years and maximum age is 50 years.

Which of the statements given above is/are correct?


A. 1 and 2 only
B. 2 and 3 only

www.insightsactivelearn.com 43
Total Marks : 200
Test-4 (Subject)
( INSTA Prelims Test Series 2021 )

C. 1 and 3 only
D. 1, 2 and 3

Correct Answer : A

Answer Justification :

Atal Pension Yojana (APY) addresses the old age income security of the working poor and the
longevity risks among the workers in unorganised sector. It encourages the workers in an
unorganised sector to voluntarily save for their retirement. The Government had
launched the scheme with effect from 1st June, 2015. The scheme replaces the
Swavalamban Yojana / NPS Lite scheme.

Hence, statement 1 is correct.

Benefits of APY:

Fixed pension for the subscribers ranging between Rs.1000 to Rs. 5000, if s/he joins and
contributes between the age of 18 years and 40 years. The contribution levels would
vary and would be low if subscriber joins early and increases if s/he joins late.
The same pension is payable to Spouse after death of Subscriber.
Return of indicative pension wealth to nominees after death of spouse.
Contributions to the Atal Pension Yojana (APY) is eligible for tax benefits
similar to the National Pension System (NPS). The tax benefits include the
additional deduction of Rs 50,000 under section 80CCD(1).

Hence, statement 2 is correct.

Atal Pension Yojana (APY) is open to all bank account holders who are not members of any
statutory social security scheme.

The minimum age of joining APY is 18 years and maximum age is 40 years. Therefore,
minimum period of contribution by the subscriber under APY would be 20 years or more.

Hence, statement 3 is incorrect.

https://economictimes.indiatimes.com/news/economy/policy/nearly-2-million-opt-for-atal-pensio
n-yojana-since-april-pfrda/articleshow/77799613.cms?from=mdr

https://vikaspedia.in/social-welfare/social-security/atal-pension-yojana

51. Consider the following statements regarding Gandhi’s Harijan Campaign


1. He had set up the All India Anti-Untouchability League.
2. He urged political workers to go to villages and work for social, economic, political and

www.insightsactivelearn.com 44
Total Marks : 200
Test-4 (Subject)
( INSTA Prelims Test Series 2021 )

cultural upliftment of the Harijans.


3. He stressed the need for caste Hindus to do ‘penance’ for untold miseries inflicted on Harijans.

Which of the statements given above is/are correct?


A. 1 and 2 only
B. 1, 2 and 3
C. 3 only
D. 1 and 3 only

Correct Answer : B

Answer Justification :

All the statements given above are correct.

Harijan Campaign

Determined to undo the divisive intentions of the government’s divide and rule policy, Gandhi
gave up all his other preoccupations and launched a whirlwind campaign against
untouchability—first from jail and then, after his release in
August 1933, from outside jail.

While in jail, he set up the All India Anti-UntouchabilityLeague in September 1932


and started the weekly Harijan in January 1933.

After his release, he shifted to the Satyagraha Ashram in Wardha as he had vowed in 1930 not
to return to Sabarmati Ashram unless swaraj was won. Starting from Wardha, he conducted a
Harijan tour of the country in the period from November 1933 to July 1934.

Covering 20,000 km, collecting money for his newly set up Harijan Sevak Sangh, and
propagating removal of untouchability in all its forms. He urged political workers to go to
villages and work for social, economic, political and cultural upliftment of the
Harijans. He undertook two fasts—on May 8 and August 16, 1934—to convince his followers
of the seriousness of his effort and the importance of the issue. These fasts created
consternation in nationalist ranks throwing many into an emotional crisis.

Throughout his Harijan tour, social work and fasts, Gandhi stressed on certain themes:

● He put forward a damning indictment of Hindu society for the kind of oppression practised
on Harijans.
● He called for total eradication of untouchability symbolised by his plea to throw open
temples to the untouchables.
● He stressed the need for caste Hindus to do ‘penance’ for untold miseries inflicted
on Harijans. For this reason he was not hostile to his critics such as Ambedkar. He said,
“Hinduism dies if untouchability lives, untouchability has to die if Hinduism is to live.”

www.insightsactivelearn.com 45
Total Marks : 200
Test-4 (Subject)
( INSTA Prelims Test Series 2021 )

52. Consider the following statements regarding the Komagata Maru Incident:
1. Komagata Maru was the name of a ship which was carrying passengers, mainly Sikh and
Punjabi Muslim.
2. It was carrying passengers from Singapore to Vancouver.
3. The ship was turned back by Canadian authorities.

Which of the statements given above is/are correct?


A. 1 and 2 only
B. 1 and 3 only
C. 2 and 3 only
D. 1, 2 and 3

Correct Answer : D

Answer Justification :

All the statements given above are correct.

The Ghadr

The Ghadr Party was a revolutionary group organized around a weekly newspaper The Ghadr
with its headquarters at San Francisco and branches along the US coast and in the Far East.

These revolutionaries included mainly ex-soldiers and peasants who had migrated from the
Punjab to the USA and Canada in search of better employment opportunities. They were based
in the US and Canadian cities along the western (Pacific) coast. Pre-Ghadr revolutionary
activity had been carried on by Ramdas Puri, G.D. Kumar, Taraknath Das, Sohan Singh Bhakna
and Lala Hardayal who reached there in 1911. To carry out revolutionary activities, the earlier
activists had set up a ‘Swadesh Sevak Home’ at Vancouver and ‘United India House’ at Seattle.
Finally, in 1913, the Ghadr was established.

Komagata Maru Incident and the Ghadr

The importance of this event lies in the fact that it created an explosive situation in the Punjab.
Komagata Maru was the name of a ship which was carrying 370 passengers, mainly
Sikh and Punjabi Muslim would-be immigrants, from Singapore to Vancouver. They
were turned back by Canadian authorities after two months of privation and
uncertainty. It was generally believed that the Canadian authorities were influenced by the
British government. The ship finally anchored at Calcutta in September 1914. The inmates
refused to board the Punjab bound train. In the ensuing conflict with the police at Budge
Budge near Calcutta, 22 persons died.

53. Consider the following statements regarding Jyotirao Phule:


1. He founded the Satyashodhak Samaj to propagate caste equality

www.insightsactivelearn.com 46
Total Marks : 200
Test-4 (Subject)
( INSTA Prelims Test Series 2021 )

2. He wrote a book named Gulamgiri.

Which of the statements given above is/are correct?


A. 1 only
B. 2 only
C. Both 1 and 2
D. Neither 1 nor 2

Correct Answer : C

Answer Justification :

Both the statements given above are correct.

One of the most vocal amongst the “low-caste” leaders was Jyotirao Phule. Born in
1827, he studied in schools set up by Christian missionaries. On growing up he developed his
own ideas about the injustices of caste society. He set out to attack the Brahmans’ claim that
they were superior to others, since they were Aryans.

He proposed that Shudras (labouring castes) and Ati Shudras (untouchables) should unite to
challenge caste discrimination. The Satyashodhak Samaj, an association Phule founded,
propagated caste equality.

In 1873, Phule wrote a book named Gulamgiri, meaning slavery. Some ten years before
this, the American Civil War had been fought, leading to the end of slavery in America. Phule
dedicated his book to all those Americans who had fought to free slaves, thus establishing a
link between the conditions of the “lower” castes in India and the black slaves in America.

54. Consider the following statements regarding Rebellion by Naval Ratings


1. It occurred in Kolkata
2. It was associated with Royal Indian Navy (RIN) ratings of HMIS Talwar
3. They protested against the use of Indian forces in Indonesia

Which of the statements given above is/are correct?


A. 1 and 3 only
B. 2 and 3 only
C. 1 and 2 only
D. 1, 2 and 3

Correct Answer : B

Answer Justification :

www.insightsactivelearn.com 47
Total Marks : 200
Test-4 (Subject)
( INSTA Prelims Test Series 2021 )

Rebellion by Naval Ratings

On February 18, 1946 some 1100 Royal Indian Navy (RIN) ratings of HMIS Talwar
went on a strike to protest against
* racial discrimination (demanding equal pay for Indian and white soldiers)
* unpalatable food
* abuse by superior officers
* arrest of a rating for scrawling ‘Quit India’ on HMIS Talwar
* INA trials
* use of Indian troops in Indonesia, demanding their withdrawal.

The rebellious ratings hoisted the tricolour, crescent, and the hammer and sickle flags on the
mast of the rebel fleet.

Other ratings soon joined and they went around Bombay in lorries holding Congress
flags threatening Europeans and policemen. Crowds brought food to the ratings and
shopkeepers invited them to take whatever they needed. Hence Statement 1 is incorrect.

55. Which of the following group of states fall under the Sixth schedule of the Indian Constitution?

A. Assam, Meghalaya, Manipur and Arunachal Pradesh


B. Arunachal Pradesh, Mizoram, Tripura and Manipur
C. Assam, Meghalaya, Tripura and Mizoram
D. Assam, Nagaland, Manipur and Tripura

Correct Answer : C

Answer Justification :

The Sixth Schedule currently includes 10 autonomous district councils in four


northeastern States — Assam, Meghalaya, Mizoram and Tripura.

Hence, option (c) is correct.

Passed by the Constituent Assembly in 1949, it seeks to safeguard the rights of tribal
population through the formation of Autonomous District Councils (ADC).

This special provision is provided under Article 244(2) and Article 275(1) of the Constitution.

Key provisions:

The governor is empowered to organise and re-organise the autonomous districts.

www.insightsactivelearn.com 48
Total Marks : 200
Test-4 (Subject)
( INSTA Prelims Test Series 2021 )

If there are different tribes in an autonomous district, the governor can divide the
district into several autonomous regions.
Composition: Each autonomous district has a district council consisting of 30 members,
of whom four are nominated by the governor and the remaining 26 are elected on the
basis of adult franchise.
Term: The elected members hold office for a term of five years (unless the council is
dissolved earlier) and nominated members hold office during the pleasure of the
governor.
Each autonomous region also has a separate regional council.
Powers of councils: The district and regional councils administer the areas under their
jurisdiction. They can make laws on certain specified matters like land, forests, canal
water, shifting cultivation, village administration, inheritance of property, marriage and
divorce, social customs and so on. But all such laws require the assent of the governor.
Village councils: The district and regional councils within their territorial jurisdictions
can constitute village councils or courts for trial of suits and cases between the tribes.
They hear appeals from them. The jurisdiction of high court over these suits and cases is
specified by the governor.
Powers and functions: The district council can establish, construct or manage primary
schools, dispensaries, markets, ferries, fisheries, roads and so on in the district. It can
also make regulations for the control of money lending and trading by non-tribals. But,
such regulations require the assent of the governor. The district and regional councils
are empowered to assess and collect land revenue and to impose certain specified taxes.

Exceptions: The acts of Parliament or the state legislature do not apply to autonomous
districts and autonomous regions or apply with specified modifications and exceptions.

The governor can appoint a commission to examine and report on any matter relating to
the administration of the autonomous districts or regions. He may dissolve a district or
regional council on the recommendation of the commission.

https://www.thehindu.com/news/national/other-states/arunachal-groups-push-for-6th-schedule-
status/article32348593.ece

https://www.insightsonindia.com/2020/08/14/arunachal-groups-push-for-6th-schedule-status/

56. Consider the following statements regarding nationalist response to British participation in the First World
War
1. Both the Moderates and Extremists supported the British in the war as a matter of duty.
2. Revolutionaries decided to utilize the opportunity to wage a war on British rule.

Which of the statements given above is/are correct?


A. 1 only

www.insightsactivelearn.com 49
Total Marks : 200
Test-4 (Subject)
( INSTA Prelims Test Series 2021 )

B. 2 only
C. Both 1 and 2
D. Neither 1 nor 2

Correct Answer : B

Answer Justification :

In the First World War (1914-1919), Britain allied with France, Russia, USA, Italy and Japan
against Germany, Austria-Hungary and Turkey. This period saw the maturing of Indian
nationalism. The nationalist response to British participation in the First World War was three-
fold:

(i) the Moderates supported the empire in the war as a matter of duty;

(ii) the Extremists, including Tilak (who was released in June 1914), supported the
war efforts in the mistaken belief that Britain would repay India’s loyalty with
gratitude in the form of self-government; Hence, statement 1 is incorrect.

(iii) the revolutionaries decided to utilize the opportunity to wage a war on British
rule and liberate the country.

Hence, statement 2 is correct.

The Indian supporters of British war efforts failed to see that the imperialist powers were
fighting to safeguard their own colonies and markets.

57. Consider the following statements


1. Gandhi held that the centre of religion must be between man and man, and not between man
and God alone, as preached by Ambedkar.
2. Gandhi held that caste system in Hinduism has nothing to do with religious precepts and
spirituality

Which of the statements given above is/are correct?


A. 1 only
B. 2 only
C. Both 1 and 2
D. Neither 1 nor 2

Correct Answer : B

Answer Justification :

Gandhi, the principal architect of the Indian freedom struggle,

www.insightsactivelearn.com 50
Total Marks : 200
Test-4 (Subject)
( INSTA Prelims Test Series 2021 )

and B.R. Ambedkar, the principal architect of the Constitution


of independent India shared many ideas, though in many ways
they held different beliefs.

The caste system and untouchability were the manifestations of the Hindu religious scriptures.
On the contrary, Gandhi held that caste system in Hinduism has nothing to do with
religious precepts and spirituality. Hence Statement 2 is correct.

Ambedkar held that the centre of religion must be between man and man, and not
between man and God alone, as preached by Gandhi. In the beginning, Ambedkar too
wanted to cast away the evil practices prevalent in Hinduism in an attempt to reform and
reconstruct, rather than destroy it fully. But in the later phase of his life, he left Hinduism,
denouncing it as an entity which couldn’t be reformed. Hence Statement 1 is incorrect.

Ambedkar denounced the Vedas and other Hindu scriptures. He believed that the Hindu
scriptures do not lend themselves to a unified and coherent understanding, and reflect strong
contradictions within and across sects. And the caste system and untouchability were the
manifestations of the Hindu religious scriptures. On the contrary, Gandhi held that caste
system in Hinduism has nothing to do with religious precepts and spirituality. For Gandhi,
caste and varna are different, and caste is perversive degeneration.

58. Consider the following pairs of Indian women social


reformers and their work:
Social Reformer: Works:
1. Sarojini Naidu Amar Jiban (My Life)
2. Rokeya Sakhawat Hossain Sultana’s Dream
3. Rashsundari Devi The Golden Threshold

Which of the pairs given above is/are correctly matched?


A. 1 and 3 only
B. 2 only
C. 1, 2 and 3
D. None

Correct Answer : B

Answer Justification :

Sarojini Naidu was an Indian political activist and poet. Naidu's work as a poet earned
her the sobriquet 'Nightingale of India' by Mahatma Gandhi. "The Golden Threshold"
was one of the book written by her.

Hence, pair 1 is incorrectly matched.

www.insightsactivelearn.com 51
Total Marks : 200
Test-4 (Subject)
( INSTA Prelims Test Series 2021 )

Rokeya Sakhawat Hossain learnt to read and write Bangla and English with the support of
her elder brother and an elder sister. She went on to become a writer. She wrote a
remarkable story titled Sultana’s Dream in 1905 to practise her English skills when
she was merely 25 years old. This story imagined a woman called Sultana who reaches
a place called Ladyland. Ladyland is a place where women had the freedom to study, work,
and create inventions like controlling rain from the clouds and flying air cars.

Hence, pair 2 is correctly matched.

Rassundari Devi is among the earliest woman writers in Bengali literature. Her
autobiography Amar Jiban (My Life) is known as the first published autobiography in
Bengali language.

Hence, pair 3 is incorrectly matched.

59. Consider the following statements regarding Home Rule League Movement
1. All India Home Rule League was established along the lines of the Russian Home Rule League.
2. In 1920, Gandhi accepted the presidentship of the All India Home Rule League.

Which of the statements given above is/are correct?


A. 1 only
B. 2 only
C. Both 1 and 2
D. Neither 1 nor 2

Correct Answer : B

Answer Justification :

Home Rule League Movement

The Home Rule Movement was the Indian response to the First World War in a less charged
but a more effective way than the response of Indians living abroad which took the form of the
romantic Ghadr adventure.

Prominent leaders—Balgangadhar Tilak, Annie Besant, G.S. Khaparde, Sir S. Subramania Iyer,
Joseph Baptista and Mohammad Ali Jinnah among others—got together and decided that it was
necessary to have a national alliance that would work throughout the year (unlike the
Congress which had annual sessions) with the main objective of demanding self-government or
home rule for all of India within the British commonwealth. This alliance was to be the All
India Home Rule League along the lines of the Irish Home Rule League. Hence,
statement 1 is incorrect.

In the end, however, two Home Rule Leagues were launched—one by Bal Gangadhar Tilak and
the other by Annie Besant, both with the aim of beginning a new trend of aggressive politics.

www.insightsactivelearn.com 52
Total Marks : 200
Test-4 (Subject)
( INSTA Prelims Test Series 2021 )

Both Tilak and Besant realized that the sanction of a Moderate-dominated Congress as well as
full cooperation of the Extremists was essential for the movement to succeed. Having failed at
the 1914 session of the Congress to reach a Moderate-Extremist rapprochement, Tilak and
Besant decided to revive political activity on their own.

In 1920, Gandhi accepted the presidentship of the All India Home Rule League, and
changed the organisation’s name to Swarajya Sabha. Within a year, however, the
league joined the Indian National Congress.

Hence, statement 2 is correct.

60. Consider the following statements regarding the Edakkal Caves:


1. It is situated in the state of Andhra Pradesh.
2. The cave is famous for having carvings belonging to the Paleolithic age.

Which of the statements given above is/are correct?


A. 1 only
B. 2 only
C. Both 1 and 2
D. Neither 1 nor 2

Correct Answer : D

Answer Justification :

None of the statements given above are correct.

Edakkal Caves are two natural caves at Edakkal, in Wayanad district of Kerala. The
caves are situated 1,200 m above sea level on Ambukutty Mala, on the Mysore
Plateau, in the Western Ghats. The Edakkal caves are believed to be camping shelters
of the Neolithic community.

The caves were discovered by Fred Fawcett, the then Superintendent of Police of the
Malabar district in 1890, during his hunting trip to Wayanad.

Edakkal caves are famous for its pictorial paintings (cave paintings), which are
considered to be of 6000 B C.

Edakkal is the only known place in India with Stone Age carvings. The carvings belong
to Neolithic and Mesolithic age. The human figures of these caves have raised hair and
some have masks. Along with these pictorial carvings, there are Tamil and Brahmi Script
in Edakkal caves.

www.insightsactivelearn.com 53
Total Marks : 200
Test-4 (Subject)
( INSTA Prelims Test Series 2021 )

https://www.thehindu.com/news/national/kerala/greens-hail-steps-to-conserve-eakkal-caves/art
icle32451101.ece

61. Consider the following statements regarding Home Rule League under Bal Gangadhar Tilak
1. Tilak set up his Indian Home Rule League before Annie Besant’s All-India Home Rule League.
2. His league was restricted to Maharashtra (excluding Bombay city), Karnataka, Central
Provinces and Berar

Which of the statements given above is/are not correct?


A. 1 only
B. 2 only
C. Both 1 and 2
D. Neither 1 nor 2

Correct Answer : D

Answer Justification :

Both the statements given above are correct.

Tilak’s League

Tilak set up his Indian Home Rule League in April 1916. Tilak held his first Home
Rule meeting at Belgaum. Poona was the headquarters of his league. His league was
restricted to Maharashtra (excluding Bombay city), Karnataka, Central Provinces and
Berar. It had six branches and the demands included swarajya, formation of linguistic
states and education in the vernacular.

Besant’s League

Annie Besant set up her All-India Home Rule League in September 1916 in Madras
(now Chennai) and covered the rest of India (including Bombay city). It had 200
branches, was loosely organised as compared to Tilak’s league and had George Arundale as
the organising secretary. Besides Arundale, the main work was done by B.W. Wadia and C.P.
Ramaswamy Aiyar.

62. Consider the following statements regarding World War II and Congress
1. Initially, Congress rejected to support world war efforts of British
2. Congress demanded to form constituent assembly to determine political structure of India
after war.
3. Subhas Chandra Bose was in the favor supporting British in war efforts

Which of the statements given above is/are correct?

www.insightsactivelearn.com 54
Total Marks : 200
Test-4 (Subject)
( INSTA Prelims Test Series 2021 )

A. 2 only
B. 1 and 3 only
C. 3 only
D. 1, 2 and 3

Correct Answer : A

Answer Justification :

World War II and Congress

Though the Congress did not like the unilateral action of the British of drawing India
into the war without consulting the Indians, it decided to support the war effort
conditionally. The hostility of the Congress to Fascism, Nazism, militarism and imperialism
had been much more consistent than the British record. Hence Statement 1 is incorrect.

The Indian offer to cooperate in the war effort had two basic conditions:
1. After the war, a constituent assembly should be convened to determine political
structure of a free India. Hence Statement 2 is correct.
2. Immediately, some form of a genuinely responsible government should be established at the
Centre. The offer was rejected by Linlithgow, the viceroy. The
Congress argued that these conditions were necessary to win public opinion for war.

Subhas Bose and other socialists, such as Acharya Narendra Dev and Jayaprakash
Narayan, who had been invited by the Congress to attend the Wardha meeting so that
different opinions could be discussed, had no sympathy for either side in the war. In
their opinion, the war was being fought by imperialists on both sides; each side wanted
to protect its colonial possessions and gain more territories to colonise, so neither side should
be supported by the nationalists. In fact, they thought it was the ideal time to launch a civil
disobedience movement, thus take advantage of the situation and snatch freedom from
Britain. Hence Statement 3 is incorrect.

63. Who among the following Viceroys had adopted a policy of patronage and “intrusive surveillance” towards
Princely states?

A. Lord Ripon
B. Lord Curzon
C. Lord Mayo
D. Lord Irwin

Correct Answer : B

Answer Justification :

www.insightsactivelearn.com 55
Total Marks : 200
Test-4 (Subject)
( INSTA Prelims Test Series 2021 )

Lord Curzon adopted a policy of patronage and “intrusive surveillance” with respect
to relations with the Princely states.

He thought the relation between the states and Government was neither feudal nor federal, a
type not based on a treaty but consisting of a series of relationships having grown under
different historical conditions that, in the course of time, gradually conformed to a single line.

Hence, option (b) is correct.

64. Which of the following joint demands was/were put forward following Lucknow Pact?
1. The term of the legislative council should be two years.
2. The salaries of the Secretary of State for India should be paid by the British treasury.
3. Half the members of the viceroy’s and provincial governors’ executive councils should be
Indians.

Select the correct answer using the code given below:


A. 1 and 2 only
B. 1 and 3 only
C. 2 and 3 only
D. 1, 2 and 3

Correct Answer : C

Answer Justification :

Lucknow Session of the Indian National Congress (1916) Readmission of Extremists


to Congress

The Lucknow session of the Indian National Congress, presided over by a Moderate,
Ambika Charan Majumdar, finally readmitted the Extremists led by Tilak to the Congress
fold.

Lucknow Pact between Congress and Muslim League

Another significant development to take place at Lucknow was the coming together of the
Muslim League and the Congress and the presentation of common demands by them to the
government. This happened at a time when the Muslim League, now dominated by the
younger militant nationalists, was coming closer to the Congress objectives and turning
increasingly anti-imperialist.

www.insightsactivelearn.com 56
Total Marks : 200
Test-4 (Subject)
( INSTA Prelims Test Series 2021 )

The Nature of the Pact

The Lucknow Pact between the Congress and the Muslim League could be considered an
important event in the course of the nationalistic struggle for freedom. While the League
agreed to present joint constitutional demands with the Congress to the government, the
Congress accepted the Muslim League’s position on separate electorates which would
continue till any one community demanded joint electorates. The Muslims were also granted a
fixed proportion of seats in the legislatures at all-India and provincial levels.

The joint demands were—

● Government should declare that it would confer self-government on Indians at an


early date.

● The representative assemblies at the central as well as provincial level should be


further expanded with an elected majority and more powers given to them.

● The term of the legislative council should be five years. Hence, statement 1 is
incorrect.

● The salaries of the Secretary of State for India should be paid by the British
treasury and not drawn from Indian funds.

Hence, statement 2 is correct.

● Half the members of the viceroy’s and provincial governors’ executive councils
should be Indians.

Hence, statement 3 is correct.

65. Panama disease, sometimes seen in news, mainly infects:

A. Maize
B. Banana
C. Mango
D. Cotton

Correct Answer : B

Answer Justification :

Panama disease, also called banana wilt, a devastating disease of bananas caused by
the soil-inhabiting fungus species Fusarium oxysporum forma specialis cubense. A
form of fusarium wilt, Panama disease is widespread throughout the tropics and can be found
wherever susceptible banana cultivars are grown.

www.insightsactivelearn.com 57
Total Marks : 200
Test-4 (Subject)
( INSTA Prelims Test Series 2021 )

Hence, option (b) is correct.

https://www.hindustantimes.com/india-news/now-banana-covid-hits-plantations/story-2ZViGgd
e0fer1eGi6BnF1N.html

66. In Indian freedom struggle, August Declaration is related to

A. Montagu’s Statement of 1917


B. Gandhiji’s call for Quit India movement.
C. Muslim league’s demand for separate electorate
D. INC’s call for Purna swaraj

Correct Answer : A

Answer Justification :

Montagu’s Statement of August 1917

The Secretary of State for India, Edwin Samuel Montagu, made a statement on August 20,
1917 in the British House of Commons in what has come to be known as the August
Declaration of 1917. The statement said: “The government policy is of an increasing
participation of Indians in every branch of administration and gradual development of self-
governing institutions with a view to the progressive realization of responsible government in
India as an integral part of the British Empire.” From now onwards, the demand by
nationalists for self-government or home rule could not be termed as seditious since
attainment of self-government for Indians now became a government policy, unlike Morley’s
statement in 1909 that the reforms were not intended to give self-government to India.

Hence, option (a) is correct.

Indian Objections

The objections of the Indian leaders to Montagu’s statement were two-fold—

(i) No specific time frame was given.

(ii) The government alone was to decide the nature and the timing of advance towards a
responsible government, and the Indians were resentful that the British would decide what
was good and what was bad for Indians.

67. Who was the Viceroy of India when World II was started?

A. Lord Willingdon
B. Lord Wavell

www.insightsactivelearn.com 58
Total Marks : 200
Test-4 (Subject)
( INSTA Prelims Test Series 2021 )

C. Lord Elgin II
D. Lord Linlithgow

Correct Answer : D

Answer Justification :

Lord Linlithgow was the viceroy of India when World War II started. Viceroy Linlithgow, in
his statement, made on October 17, 1939, tried to use the Muslim League and the princes
against the Congress.

68. Which of the following were drawbacks of Government of India Act, 1919?
1. Franchise was very limited.
2. At the center, the legislature had no control over the viceroy and his executive council.
3. The provincial ministers had no control over finances and over the bureaucrats.

Select the correct answer using the code given below:


A. 1 only
B. 1 and 3 only
C. 2 and 3 only
D. 1, 2 and 3

Correct Answer : D

Answer Justification :

All the statements given above are correct.

Montagu-Chelmsford Reforms and Government of India Act, 1919:

In line with the government policy contained in Montagu’s statement of August 1917, the
government announced further constitutional reforms in July 1918, known as Montagu-
Chelmsford or Montford Reforms. Based on these, the Government of India Act, 1919 was
enacted.

Drawbacks

The reforms had many drawbacks—

(i) Franchise was very limited. The electorate was extended to some one-and-a-half million
for the central legislature, while the population of India was around 260 million, as per one
estimate.

(ii) At the centre, the legislature had no control over the viceroy and his executive

www.insightsactivelearn.com 59
Total Marks : 200
Test-4 (Subject)
( INSTA Prelims Test Series 2021 )

council.

(iii) Division of subjects was not satisfactory at the centre.

(iv) Allocation of seats for central legislature to the provinces was based on ‘importance’ of
provinces—for instance, Punjab’s military importance and Bombay’s commercial importance.

(v) At the level of provinces, division of subjects and parallel administration of two parts was
irrational and, hence, unworkable. Subjects like irrigation, finance, police, press and justice
were ‘reserved’.

(vi) The provincial ministers had no control over finances and over the bureaucrats;
this would lead to constant friction between the two. Ministers were often not consulted on
important matters too; in fact, they could be overruled by the governor on any matter that the
latter considered special.

69. Who among the following leaders linked capitalism with imperialism by giving the statement 'Imperialism and
militarism are the twin children of capitalism'?

A. C.R. Das
B. Motilal Nehru
C. Lala Lajpat Rai
D. Sir Pherozeshah Mehta

Correct Answer : C

Answer Justification :

The trade union movement was led by All India Trade Union Congress (AITUC)
founded in 1920. Lala Lajpat Rai was its first president and Dewan Chaman Lal its
general secretary.

Lajpat Rai was the first to link capitalism with imperialism. He stated that
“imperialism and militarism are the twin children of capitalism”.

Hence, option (c) is correct.

70. The Oromo community, recently seen in news, primarily belong to:

A. Australia
B. Ethiopia
C. Egypt
D. Brazil

www.insightsactivelearn.com 60
Total Marks : 200
Test-4 (Subject)
( INSTA Prelims Test Series 2021 )

Correct Answer : B

Answer Justification :

The Oromo community is the largest ethnic group in Ethiopia.

Oromos speak the Oromo language as their mother tongue (also called Afaan Oromoo and
Oromiffa), which is part of the Cushitic branch of the Afroasiatic language family.

Hence, option (b) is correct.

https://indianexpress.com/article/explained/hachalu-hundessa-ethiopia-protests-6489875/

71. Consider the following statements regarding the Tolstoy Farm


1. The Tolstoy Farm was founded in 1910 and named after the Russian writer and moralist
2. The Tolstoy Farm was the first farm to be established by Gandhi.
3. Co-educational classes were held, and boys and girls were encouraged to work together.

Which of the statements given above is/are correct?


A. 1 only
B. 1 and 3 only
C. 2 and 3 only
D. 1, 2 and 3

Correct Answer : B

Answer Justification :

Tolstoy Farm

As it became rather difficult to sustain the high pitch of the struggle, Gandhi decided to devote
all his attention to the struggle. The Tolstoy Farm was founded in 1910 and named as
such by Gandhi’s associate, Herman Kallenbach, after the Russian writer and
moralist, whom Gandhi admired and corresponded with. Besides being an experiment in
education, it was to house the families of the satyagrahis and to give them a way to sustain
themselves.

Hence, statement 1 is correct.

The Tolstoy Farm was the second of its kind established by Gandhi. Hence, statement
2 is incorrect. He had set up the Phoenix Farm in 1904 in Natal, inspired by a reading
of John Ruskin’s Unto This Last, a critique of capitalism, and a work that extolled the
virtues of the simple life of love, labour, and the dignity of human beings. As at the Phoenix

www.insightsactivelearn.com 61
Total Marks : 200
Test-4 (Subject)
( INSTA Prelims Test Series 2021 )

settlement, at Tolstoy Farm too, manual work went hand-in-hand with instruction. Vocational
training was introduced to give “all-round development to the boys and girls”.

Co-educational classes were held, and boys and girls were encouraged to work
together. Hence, statement 3 is correct.

The activities included general labour, cooking, scavenging, sandal making, simple carpentry
and messenger work. Manual work such as sweeping, scavenging and fetching water was
perceived to be invaluable to the psychological, social and moral well-being of an integrated
community. Gandhi’s objective in this context was to inculcate the ideals of social service and
citizenship besides a healthy respect for manual work from the early formative years itself.
The farm worked till 1913.

72. Which of the following was/were the proposals of August Offer?


1. Dominion status to India
2. Constitution to be adopted with the consent of minorities.
3. Partition of India into two dominions

Select the correct answer using the code given below


A. 1 and 2 only
B. 3 only
C. 1 and 3 only
D. 1, 2 and 3

Correct Answer : A

Answer Justification :

The government came up with its own offer to get the cooperation of India in the war effort.
Linlithgow announced the August Offer (August 1940) which proposed:

● Dominion status as the objective for India; Hence Statement 1 is correct.


● expansion of viceroy’s executive council which would have a majority of Indians (who would
be drawn from major political parties);
● setting up of a constituent assembly after the war where mainly Indians would decide the
constitution according to their social, economic and political conceptions, subject to fulfilment
of the obligation of the government regarding defence, minority rights, treaties with States, all
India services; and
● no future constitution to be adopted without the consent of minorities. Hence
Statement 2 is correct.

The government had taken the adamant position that no constitutional advance could be made
till the Congress came to an agreement with the Muslim leaders. It issued ordinance after
ordinance taking away the freedom of speech and that of the press and the right to organise

www.insightsactivelearn.com 62
Total Marks : 200
Test-4 (Subject)
( INSTA Prelims Test Series 2021 )

associations.

Partition of India into two dominions was not in August Proposals. Hence Statement
3 is incorrect.

73. Consider the following events:

1. Wavell Plan
2. Cripps Mission
3. Quit India Movement
4. Cabinet Mission

The correct chronological sequence of these events:


A. 2-3-1-4
B. 2-3-4-1
C. 3-2-1-4
D. 3-2-4-1

Correct Answer : A

Answer Justification :

Cripps Mission: March, 1942

The Cripps Mission was a failed attempt in late March 1942 by the British government to
secure full Indian cooperation and support for their efforts in World War II. The mission was
headed by a senior minister Sir Stafford Cripps.

Quit India Movement: August 1942

The Quit India Movement (translated into several Indian languages as the Leave India
Movement), also known as the August Movement, was a movement launched at the Bombay
session of the All-India Congress Committee by Mahatma Gandhi on 8 August 1942, during
World War II, demanding an end to British rule of India.

Wavell Plan: June, 1945

The Simla Conference of 1945 was a meeting between the Viceroy of India Lord Wavell and
the major political leaders of British India at Viceregal Lodge in Simla. Convened to agree on
and approve the Wavell Plan for Indian self-government, and there it reached a potential
agreement for the self-rule of India that provided separate representation for Muslims and
reduced majority powers for both communities in their majority regions.

Cabinet Mission: March, 1946

The Cabinet Mission came to India with an to discuss the transfer of powers from the British

www.insightsactivelearn.com 63
Total Marks : 200
Test-4 (Subject)
( INSTA Prelims Test Series 2021 )

government to the Indian leadership, with the aim of preserving India's unity and granting its
independence.

74. Who is the person to call Mahatma Gandhi as “Father of Nation”?

A. Jawaharlal Nehru
B. Subhas Chandra Bose
C. Jayprakash Narayan
D. Sardar Vallabhbhai Patel

Correct Answer : B

Answer Justification :

On July 6, 1944, Subhas Bose addressed Mahatma Gandhi as ‘Father of Nation’—from


the Azad Hind Radio (the first person to call Gandhi, ‘Father of Nation’). Subhas Bose
asked for Gandhi’s blessings for “India’s last war of independence”.

75. Consider the following statements regarding Mahatma Ayyankali:


1. He is a social reformer who fought against the caste discrimination.
2. Mahatma Gandhi had called him as ‘Pulaya Raja’.
3. He started Sadhu Jana Paripalana Sangham (association for the protection of the poor).

Which of the statements given above is/are correct?


A. 1 and 2 only
B. 2 and 3 only
C. 1 and 3 only
D. 1, 2 and 3

Correct Answer : D

Answer Justification :

All the statements given above are correct.

Mahatma Ayyankali:

Born on August 28th, 1863 in a small village in the princely state of Travancore, now within
the south of the modern-day nation of Kerala,

The caste discrimination he faced as a child turned him into a leader of an anti-caste
movement and who later fought for basic rights including access to public spaces and entry to

www.insightsactivelearn.com 64
Total Marks : 200
Test-4 (Subject)
( INSTA Prelims Test Series 2021 )

schools.

Mahatma Gandhi called Ayyankali as ‘Pulaya king’. Indira Gandhi described him as
‘India’s greatest son’.

His contributions to Dalit developments:

Ayyankali in 1893 rode an ox-cart challenging the ‘ban’ on untouchables from accessing
public roads by caste-Hindus.
He also led a rally to assert the rights of ‘untouchables’ at Balaramapuram. The walk
Ayyankali took came to be known as ‘walk for freedom’ and the consequent riots as
‘Chaliyar riots’.
Ayyankali efforts influenced many changes that improved social wellbeing of those
people, who are today referred to as Dalits.
Ayyankali became a stated protestor for Pulayar rights. Because of the protests led
through Ayyankali, in 1907 a decree turned into issued to confess students from the
untouchable network to government schools.
Inspired by Sree Narayana Guru, a social reformer from Ezhava caste, Ayyankali
started Sadhu Jana Paripalana Sangham (association for the protection of the
poor) which later raised funds to start their own schools.

https://www.outlookindia.com/newsscroll/social-reformer--ayyankalis-birth-anniversary-celebra
ted/1924797

https://www.insightsonindia.com/2020/08/29/mahatma-ayyankali/

76. Consider the following statements regarding the Champaran Satyagraha


1. Gandhi was requested by Rajkumar Shukla, to look into the problems of the farmers in context
of indigo planters.
2. The European planters were forcing Tinkathia system on peasants.
3. As a compromise with the planters, Gandhi agreed that only 50 per cent of the money taken
should be compensated.

Which of the statements given above is/are correct?


A. 1 and 2 only
B. 1 and 3 only
C. 2 and 3 only
D. 1, 2 and 3

Correct Answer : A

Answer Justification :

Champaran Satyagraha (1917)—First Civil Disobedience

www.insightsactivelearn.com 65
Total Marks : 200
Test-4 (Subject)
( INSTA Prelims Test Series 2021 )

Gandhi was requested by Rajkumar Shukla, a local man, to look into the problems of
the farmers in context of indigo planters of Champaran in Bihar. The European
planters had been forcing the peasants to grow indigo on 3/20 part of the total land
(called tinkathia system).

Hence, statement 1 and statement 2 are correct.

When towards the end of the nineteenth century German synthetic dyes replaced indigo, the
European planters demanded high rents and illegal dues from the peasants in order to
maximise their profits before the peasants could shift to other crops. Besides, the peasants
were forced to sell the produce at prices fixed by the Europeans.

When Gandhi, joined now by Rajendra Prasad, Mazharul-Haq, Mahadeo Desai, Narhari Parekh,
and J.B. Kripalani, reached Champaran to probe into the matter, the authorities ordered him to
leave the area at once. Gandhi defied the order and preferred to face the punishment. This
passive resistance or civil disobedience of an unjust order was a novel method at that time.
Finally, the authorities retreated and permitted Gandhi to make an enquiry. Now, the
government appointed a committee to go into the matter and nominated Gandhi as a member.
Gandhi was able to convince the authorities that the tinkathia system should be abolished and
that the peasants should be compensated for the illegal dues extracted from them. As a
compromise with the planters, he agreed that only 25 per cent of the money taken
should be compensated. Hence, statement 3 is incorrect.

77. Consider the following statements regarding Individual Satyagraha


1. Vinoba Bhave was the first to offer the satyagraha
2. The demand of the satyagrahi would be the freedom of speech against the war through an
anti-war declaration.

Which of the statements given above is/are not correct?


A. 1 only
B. 2 only
C. Both 1 and 2
D. Neither 1 nor 2

Correct Answer : D

Answer Justification :

Both the statements are correct.

The aims of launching individual satyagraha were — (i) to show that nationalist patience was
not due to weakness;

(ii) to express people’s feeling that they were not interested in the war and that they made no
distinction between Nazism and the double autocracy that ruled India; and

www.insightsactivelearn.com 66
Total Marks : 200
Test-4 (Subject)
( INSTA Prelims Test Series 2021 )

(iii) to give another opportunity to the government to accept Congress’


demands peacefully.

The demand of the satyagrahi would be the freedom of speech against the war
through an anti-war declaration.

If the government did not arrest the satyagrahi, he or she would not only repeat it but move
into villages and start a march towards Delhi, thus precipitating a movement which came to be
known as the ‘Delhi Chalo Movement’.

Vinoba Bhave was the first to offer the satyagraha and Nehru, the second. By May
1941, 25,000 people had been convicted for individual civil disobedience.

78. Which of the following Wildlife Sanctuaries/National Parks is/are located in Goa?
1. Bhagwan Mahaveer Sanctuary
2. Mahatma Gandhi National Park
3. Mollem National Park

Select the correct answer using the code given below


A. 3 only
B. 1 and 2 only
C. 2 only
D. 1 and 3 only

Correct Answer : D

Answer Justification :

Bhagwan Mahaveer Sanctuary and Mollem National Park is a 240 square kilometres (93
sq mi) protected area located in the Western Ghats of South India, in Sanguem taluk, Goa
State, along the eastern border with Karnataka. The area is situated near the town of Molem,
57 kilometres (35 mi) east of Panaji, the state capital of Goa. Hence Statement 1 and 3 are
correct.

Other protected areas in Goa

Dr Salim Ali Bird Sanctuary


Cotigao Wildlife Sanctuary
Bondla Wildlife Sanctuary
Mhadei Wildlife Sanctuary
Bhagwan Mahaveer Sanctuary
Mollem National Park

Mahatma Gandhi Marine National Park is a national park of India near Wandoor on
the Andaman Islands. It belongs to the South Andaman administrative district, part of the

www.insightsactivelearn.com 67
Total Marks : 200
Test-4 (Subject)
( INSTA Prelims Test Series 2021 )

Indian union territory of Andaman and Nicobar Islands. Hence Statement 2 is incorrect.

79. Consider the following statements regarding the Rowlatt Act


1. The committee had recommended that activists should be deported or imprisoned without trial
for lifetime.
2. It allowed arrest of Indians without warrant on the mere suspicion of ‘treason’.

Which of the statements given above is/are correct?


A. 1 only
B. 2 only
C. Both 1 and 2
D. Neither 1 nor 2

Correct Answer : B

Answer Justification :

The Rowlatt Act

Just six months before the Montford Reforms were to be put into effect, two bills were
introduced in the Imperial Legislative Council. One of them was dropped, but the other—an
extension to the Defence of India Regulations Act 1915—was passed in March 1919. It was
what was officially called the Anarchical and Revolutionary Crimes Act, but popularly known
as the Rowlatt Act. It was based on the recommendations made in the previous year to the
Imperial Legislative Council by the Rowlatt Commission, headed by the British judge, Sir
Sidney Rowlatt, to investigate the ‘seditious conspiracy’ of the Indian people. (The committee
had recommended that activists should be deported or imprisoned without trial for
two years, and that even possession of seditious newspapers would be adequate
evidence of guilt.) Hence, statement 1 is incorrect.

The act allowed political activists to be tried without juries or even imprisoned
without trial. It allowed arrest of Indians without warrant on the mere suspicion of
‘treason’. Such suspects could be tried in secrecy without recourse to legal help. A special
cell consisting of three high court judges was to try such suspects and there was no court of
appeal above that panel. This panel could even accept evidence not acceptable under the
Indian Evidences Act. The law of habeas corpus, the basis of civil liberty, was sought to be
suspended.

Hence, statement 2 is correct.

80. Consider the following statements regarding the Sovereign Gold Bond (SGB) Scheme:
1. Sovereign Gold Bonds (SGB) are government securities denominated in grams of gold.
2. The Bond is issued by the India Bullion and Jewellers Association Ltd. (IBJA) on behalf of

www.insightsactivelearn.com 68
Total Marks : 200
Test-4 (Subject)
( INSTA Prelims Test Series 2021 )

Government of India.
3. Persons resident in India as defined under Foreign Exchange Management Act, 1999 are
eligible to invest in SGB.

Which of the statements given above is/are correct?


A. 1 only
B. 2 and 3 only
C. 1 and 3 only
D. 1, 2 and 3

Correct Answer : C

Answer Justification :

Sovereign Gold Bond (SGB) Scheme:

SGBs are government securities denominated in grams of gold. They are substitutes
for holding physical gold. Investors have to pay the issue price in cash and the bonds
will be redeemed in cash on maturity.

Hence, statement 1 is correct.

The Bond is issued by Reserve Bank on behalf of Government of India.

Hence, statement 2 is incorrect.

Persons resident in India as defined under Foreign Exchange Management Act, 1999
are eligible to invest in SGB. Eligible investors include individuals, HUFs, trusts,
universities, charitable institutions, etc.

Hence, statement 3 is correct.

The Bonds bear interest at the rate of 2.50 per cent (fixed rate) per annum on the amount of
initial investment. Interest will be credited semiannually to the bank account of the investor
and the last interest will be payable on maturity along with the principal.

The Reserve Bank of India, in consultation with the Government of India, issues tranches of
Sovereign Gold Bonds.

The Bonds will be sold through Scheduled Commercial banks(except Small Finance
Banks and Payment Banks), Stock Holding Corporation of India Limited (SHCIL),
designated post offices, and recognised stock exchanges viz., National Stock Exchange of India
Limited and Bombay Stock Exchange Limited.

Benefits:

The Sovereign Gold Bonds will be available both in demat and paper form.
The tenor of the bond is for a minimum of 8 years with option to exit from 5th year.

www.insightsactivelearn.com 69
Total Marks : 200
Test-4 (Subject)
( INSTA Prelims Test Series 2021 )

They will carry sovereign guarantee both on the capital invested and the interest.
Bonds can be used as collateral for loans.
Bonds would be allowed to be traded on exchanges to allow early exits for investors who
may so desire.
The Capital gain tax arising on redemption of SGB to an individual is exempted.

https://www.hindustantimes.com/business-news/sovereign-gold-bond-to-open-for-subscription-t
oday-issue-price-fixed-at-rs-5-334-per-gram/story-cqY16RmgkbPLqGcg8v26zM.html

https://vikaspedia.in/social-welfare/financial-inclusion/swarnabharath

81. Consider the following statements regarding the Khilafat issue


1. Khilafat Committee was formed in 1919 under the leadership of the Ali brothers.
2. Muhammad Ali Jinnah was the president of the All India Khilafat Committee.

Which of the statements given above is/are correct?


A. 1 only
B. 2 only
C. Both 1 and 2
D. Neither 1 nor 2

Correct Answer : A

Answer Justification :

The Khilafat issue paved the way for the consolidation of the emergence of a radical nationalist
trend among the younger generation of Muslims and the section of traditional Muslim scholars
who were becoming increasingly critical of British rule.

In India, too, the Muslims demanded from the British

(i) that the Khalifa’s control over Muslim sacred places should be retained, and

(ii) the Khalifa should be left with sufficient territories after territorial arrangements.

In early 1919, a Khilafat Committee was formed under the leadership of the Ali
brothers (Shaukat Ali and Muhammad Ali), Maulana Azad, Ajmal Khan and Hasrat
Mohani, to force the British government to change its attitude towards Turkey. Thus,
the ground for a country-wide agitation was prepared.

Hence, statement 1 is correct.

The All India Khilafat Conference held in Delhi in November 1919, a call was made for the
boycott of British goods. The Khilafat leaders also clearly spelt out that unless peace terms
after the War were favourable to Turkey they would stop all cooperation with the Government.
Gandhi, who was the president of the All India Khilafat Committee, saw in the issue a

www.insightsactivelearn.com 70
Total Marks : 200
Test-4 (Subject)
( INSTA Prelims Test Series 2021 )

platform from which mass and united noncooperation could be declared against the
Government. Hence, statement 2 is incorrect.

82. Quit India Movement was launched in response to

A. Cabinet Mission Plan


B. Cripps Proposals
C. Simon Commission Report
D. Wavell Plan

Correct Answer : B

Answer Justification :

The Quit India Movement (translated into several Indian languages as the Leave India
Movement), also known as the August Movement, was a movement launched at the Bombay
session of the All-India Congress Committee by Mahatma Gandhi on 8 August 1942, during
World War II, demanding an end to British rule of India.

After the failure of the Cripps Mission to secure Indian support for the British war
effort, Gandhi made a call to Do or Die in his Quit India speech delivered in Bombay
on 8 August 1942 at the Gowalia Tank Maidan.

83. Which of the following major decisions were taken at the Lahore session?
1. Gandhi-Irwin pact was endorsed.
2. January 26, 1930 was fixed as the first Independence (Swarajya) Day, to be celebrated
everywhere.
3. Complete independence was declared as the aim of the Congress.

Select the correct answer using the code given below:


A. 1 only
B. 1 and 3 only
C. 2 and 3 only
D. 1, 2 and 3

Correct Answer : C

Answer Justification :

Lahore Congress and Purna Swaraj

Jawaharlal Nehru, who had done more than anyone else to popularise the concept of purna

www.insightsactivelearn.com 71
Total Marks : 200
Test-4 (Subject)
( INSTA Prelims Test Series 2021 )

swaraj, was nominated the president for the Lahore session of the Congress (December 1929)
mainly due to Gandhi’s backing (15 out of 18 Provincial Congress Committees had opposed
Nehru).

The following major decisions were taken at the Lahore session.

● The Round Table Conference was to be boycotted.

● Complete independence was declared as the aim of the Congress. Hence, statement
2 is correct.

● Congress Working Committee was authorised to launch a programme of civil


disobedience including non-payment of taxes and all members of legislatures were
asked to resign their seats.

● January 26, 1930 was fixed as the first Independence (Swarajya) Day, to be
celebrated everywhere.

Hence, statement 3 is correct.

The 'Gandhi - Irwin Pact' was a political agreement signed by Mahatma Gandhi and
Lord Irwin, Viceroy of India, on 5 March 1931 before the second Round Table
Conference in London.

Hence, statement 1 is incorrect.

84. Pradhan Mantri Vaya Vandana Yojana is implemented by which of the following ministry/agency?

A. Ministry of Social Justice


B. Ministry of Finance
C. Ministry of Rural Development
D. NITI Ayog

Correct Answer : B

Answer Justification :

Pradhan Mantri Vaya Vandana Yojana (PMVVY)

Based on the success and popularity of Varishtha Pension Bima Yojana 2003
(VPBY-2003), Varishtha Pension Bima Yojana 2014 (VPBY-2014) schemes, and to
protect elderly persons aged 60 years and above against a future fall in their interest income
due to the uncertain market conditions, as also to provide social security during old age and
being implemented through Life Insurance Corporation (LIC) of India.

As per the scheme, on payment of an initial lump sum amount ranging from a minimum

www.insightsactivelearn.com 72
Total Marks : 200
Test-4 (Subject)
( INSTA Prelims Test Series 2021 )

purchase price of Rs. 1, 50,000/- for a minimum pension of Rs 1000/- per month to a maximum
purchase price of Rs. 7, 50,000/- for maximum pension of Rs. 5,000/- per month, subscribers
will get an assured pension based on a guaranteed rate of return of 8% per annum, payable
monthly.

The scheme is implemented by Ministry of Finance.

85. Namdapha National Park, sometimes seen in news, is located in:

A. Assam
B. Arunachal Pradesh
C. Himachal Pradesh
D. Sikkim

Correct Answer : B

Answer Justification :

Namdapha National Park is a 1,985 km² large protected area in Arunachal Pradesh of
Northeast India. With more than 1,000 floral and about 1,400 faunal species, it is a
biodiversity hotspot in the Eastern Himalayas.

Hence, option (b) is correct.

https://www.thehindu.com/news/national/japanese-found-chinese-butterfly-is-now-indian/articl
e31963607.ece

86. Who among the following was/were the members of Congress-Khilafat Swarajya Party?
1. C. Rajagopalachari
2. C.R. Das
3. Motilal Nehru
4. Rajendra Prasad

Select the correct answer using the code given below:


A. 1 and 2 only
B. 2 and 3 only
C. 3 and 4 only
D. 1 and 4 only

Correct Answer : B

Answer Justification :

www.insightsactivelearn.com 73
Total Marks : 200
Test-4 (Subject)
( INSTA Prelims Test Series 2021 )

Genesis of Congress-Khilafat Swarajya Party

After Gandhi’s arrest (March 1922), there was disintegration, disorganisation and
demoralisation among nationalist ranks. A debate started among Congressmen on what to do
during the transition period, i.e., the passive phase of the movement. One section led by C.R.
Das, Motilal Nehru and Ajmal Khan wanted an end to the boycott of legislative councils so that
the nationalists could enter them to expose the basic weaknesses of these assemblies and use
these councils as an arena of political struggle to arouse popular enthusiasm. They wanted, in
other words, to ‘end or mend’ these councils, i.e., if the government did not respond to the
nationalists’ demands, then they would obstruct the working of these councils.

Those advocating entry into legislative councils came to be known as the ‘Swarajists’,
while the other school of thought led by C. Rajagopalachari, Vallabhbhai Patel,
Rajendra Prasad and M.A. Ansari came to be known as the ‘Nochangers’.

The ‘No-changers’ opposed council entry, advocated concentration on constructive work, and
continuation of boycott and non-cooperation, and quiet preparation for resumption of the
suspended civil disobedience programme. The differences over the question of council entry
between the two schools of thought resulted in the defeat of the Swarajists’ proposal of
‘ending or mending’ the councils at the Gaya session of the Congress (December1922). C.R.
Das and Motilal Nehru resigned from the presidentship and secretaryship respectively of the
Congress and announced the formation of Congress-Khilafat Swarajya Party or simply
Swarajist Party, with C.R. Das as the president and Motilal Nehru as one of the
secretaries.

Hence, option (b) is correct.

87. Consider the following statements regarding Quit India Movement


1. The Quit India Resolution was ratified at the Congress meeting at Gowalia Tank, Calcutta.
2. It resolved to demand an immediate end to British rule in India.
3. Jawaharlal Nehru was named as the leader of the movement

Which of the statements given above is/are correct?


A. 1 and 2 only
B. 2 only
C. 3 only
D. 2 and 3 only

Correct Answer : B

Answer Justification :

The Quit India Resolution was ratified at the Congress meeting at Gowalia Tank, Bombay, on

www.insightsactivelearn.com 74
Total Marks : 200
Test-4 (Subject)
( INSTA Prelims Test Series 2021 )

August 8, 1942. Hence Statement 1 is incorrect.

The meeting also resolved to

● demand an immediate end to British rule in India.

● declare commitment of free India to defend itself against all types of Fascism and
imperialism.

● form a provisional Government of India after British withdrawal.

● sanction a civil disobedience movement against British rule.

Gandhi was named the leader of the struggle. Hence Statement 3 is incorrect.

88. Consider the following statements regarding Mount Harriet National Park
1. It is located in Andaman and Nicobar Islands
2. Tropical Evergreen forest is the dominant vegetation of this park

Which of the statements given above is/are correct?


A. 1 only
B. 2 only
C. Both 1 and 2
D. Neither 1 nor 2

Correct Answer : C

Answer Justification :

Mount Harriet National Park is located in the south of the Andaman and Nicobar islands.
Mount Harriet is the third-highest peak in the Andaman and Nicobar archipelago next to
Saddle

The park is covered with evergreen forest pockets. It is rich in flora and faunal species like
Andaman wild pigs, saltwater crocodiles, butterflies and palm trees.

https://en.wikipedia.org/wiki/Mount_Harriet_National_Park

89. Consider the following statements regarding Socialist Ideas in India in British India:
1. The Communist Party of India (CPI) was formed in 1920 in Tashkent by M.N. Roy.
2. In 1925, the Indian Communist Conference at Kanpur formalized the foundation of the CPI.

Which of the statements given above is/are correct?


A. 1 only

www.insightsactivelearn.com 75
Total Marks : 200
Test-4 (Subject)
( INSTA Prelims Test Series 2021 )

B. 2 only
C. Both 1 and 2
D. Neither 1 nor 2

Correct Answer : C

Answer Justification :

Both the statements given above are correct.

Spread of Marxist and Socialist Ideas

Ideas of Marx and Socialist thinkers inspired many groups to come into existence as socialists
and communists. These ideas also resulted in the rise of a left wing within the Congress,
represented by Jawaharlal Nehru and Subhash Chandra Bose.

The Communist Party of India (CPI) was formed in 1920 in Tashkent (now, the capital
of Uzbekistan) by M.N. Roy, Abani Mukherji and others after the second Congress of
Commintern. M.N. Roy was also the first to be elected to the leadership of
Commintern.

In 1924, many communists—S.A. Dange, Muzaffar Ahmed, Shaukat Usmani, Nalini


Gupta—were jailed in the Kanpur Bolshevik Conspiracy Case.

In 1925, the Indian Communist Conference at Kanpur formalized the foundation of


the CPI. In 1929, the government crackdown on communists resulted in the arrest and trial of
31 leading communists, trade unionists and left-wing leaders; they were tried at Meerut in the
famous Meerut conspiracy case.

Workers’ and peasants’ parties were organized all over the country and they propagated
Marxist and communist ideas.

All these communist groups and workers’ and peasants’ parties remained an integral part of
the national movement and worked along with the Congress.

90. Consider the following statements regarding Geographical Indication (GI) tag
1. The registration of a geographical indication is valid for a period of 10 years.
2. It is used for agricultural product, manufactured products and services

Which of the statements given above is/are correct?


A. 1 only
B. 2 only
C. Both 1 and 2
D. Neither 1 nor 2

www.insightsactivelearn.com 76
Total Marks : 200
Test-4 (Subject)
( INSTA Prelims Test Series 2021 )

Correct Answer : A

Answer Justification :

GI Tag is name or sign used on certain products which correspond to specific


geographical location or origin. It is used for agricultural, natural and manufactured
goods having special quality and established reputation. Hence Statement 2 is
incorrect.

The goods and products having tag are recognised for their origin, quality and reputation and
gives it required edge in global market. It also ensures that none can use their name, giving
them exclusivity.

GI Tag Governance

India, as a member of the World Trade Organization (WTO), enacted the Geographical
Indications of Goods (Registration and Protection) Act, 1999 has come into force with effect
from 15 September 2003.

This Act is administered by Controller General of Patents. Designs and Trade Marks, who is
also Registrar of Geographical Indications which works under Ministry of Commerce and is
based in Chennai. Tamil Nadu.

The registration of GI Tag is valid for 10 years after which it needs to be renewed.
Violation of GI tags is punishable offence under law. The purpose of GI tag enables
stakeholders to authenticate their production while earning a premium and derive an
improved livelihood. Hence Statement 1 is correct.

The GI tag ensures that none other than those registered as authorised users (or at least those
residing inside the geographic territory) are allowed to use the popular product name.
Darjeeling tea became the first GI tagged product in India.

91. Consider the following statements regarding the Hindustan Republican Association
1. The HRA was founded by Ramprasad Bismil, Jogesh Chandra Chatterjee and Sachin Sanyal
2. Its aim was to establish the Federal Republic of United States of India whose basic principle
would be adult franchise.
3. Under the leadership of Chandra Shekhar Azad, the name of HRA was changed to Hindustan
Socialist Republican Association (HSRA).

Which of the statements given above is/are correct?


A. 1 only
B. 1 and 3 only
C. 2 and 3 only
www.insightsactivelearn.com 77
Total Marks : 200
Test-4 (Subject)
( INSTA Prelims Test Series 2021 )

D. 1, 2 and 3

Correct Answer : D

Answer Justification :

All the statements given above are correct.

The revolutionary activity in this region was dominated by the Hindustan Republican
Association/Army or HRA (later renamed Hindustan Socialist Republican Association or
HSRA). The HRA was founded in October 1924 in Kanpur by Ramprasad Bismil, Jogesh
Chandra Chatterjee and Sachin

Sanyal, with an aim to organize an armed revolution to overthrow the colonial government and
establish in its place the Federal Republic of United States of India whose basic principle
would be adult franchise.

The HSRA

Determined to overcome the Kakori setback, the younger revolutionaries, inspired by socialist
ideas, set out to reorganize Hindustan Republic Association at a historic meeting in the ruins
of Ferozshah Kotla in Delhi (September 1928). Under the leadership of Chandra Shekhar
Azad, the name of HRA was changed to Hindustan Socialist Republican Association
(HSRA). The participants included Bhagat Singh, Sukhdev, Bhagwaticharan Vohra from
Punjab and Bejoy Kumar Sinha, Shiv Verma and Jaidev Kapur from the United Provinces. The
HSRA decided to work under a collective leadership and adopted socialism as its official goal.

The famous statement of the revolutionary position is contained in the book The Philosophy of
the Bomb written by Bhagwaticharan Vohra.

92. Consider the following statements regarding Rajagopalachari Formula


1. Gandhi rejected the Rajagopalachari formula
2. According to formula, Muslim league would cooperate with Congress in forming a provisional
government at centre.

Which of the statements given above is/are correct?


A. 1 only
B. 2 only
C. Both 1 and 2
D. Neither 1 nor 2

Correct Answer : B

Answer Justification :

www.insightsactivelearn.com 78
Total Marks : 200
Test-4 (Subject)
( INSTA Prelims Test Series 2021 )

Rajagopalachari Formula

C. Rajagopalachari (CR), the veteran Congress leader, prepared a formula for Congress-
League cooperation in 1944. It was a tacit acceptance of the League’s demand for Pakistan.
Gandhi supported the formula. Hence Statement 1 is incorrect.

The main points in the CR Plan were:

● Muslim League to endorse Congress demand for independence.


● League to cooperate with Congress in forming a provisional government at centre. Hence
Statement 2 is correct.
● After the end of the war, the entire population of Muslim majority areas in the North-West
and North-East India to decide by a plebiscite, whether or not to form a separate sovereign
state.
● In case of acceptance of partition, agreement to be made jointly for safeguarding defence,
commerce, communications, etc.
● The above terms to be operative only if England transferred full powers to India.

93. Consider the following statements regarding The Banks Board Bureau (BBB)
1. It is an autonomous recommendatory body.
2. The Banks Board Bureau is a public authority as defined in the Right to Information Act, 2005.

Which of the statements given above is/are correct?


A. 1 only
B. 2 only
C. Both 1 and 2
D. Neither 1 nor 2

Correct Answer : C

Answer Justification :

Both the statements are correct.

Bank Board Bureau is an autonomous recommendatory body. The Ministry of Finance


takes the final decision on the appointments in consultation with the Prime Minister’s Office.

Functions:

1. Apart from recommending personnel for the PSBs, the Bureau has also been assigned
with the task of recommending personnel for appointment as directors in government-
owned insurance companies.
2. It engages with the board of directors of all the public sector banks to formulate

www.insightsactivelearn.com 79
Total Marks : 200
Test-4 (Subject)
( INSTA Prelims Test Series 2021 )

appropriate strategies for their growth and development.


3. It is tasked with improving corporate governance at public sector banks, building
capacities, etc.

The Banks Board Bureau is a public authority as defined in the Right to Information
Act, 2005.

94. Consider the following statements regarding Indian Army (INA)


1. Mohan Singh created an army out of the Indian prisoners of war (POWs)
2. INA had decided to support the national cause only on the invitation of Indian National
Congress

Which of the statements given above is/are correct?


A. 1 only
B. 2 only
C. Both 1 and 2
D. Neither 1 nor 2

Correct Answer : C

Answer Justification :

Both the statements are correct.

Origin and First Phase of the Indian National Army

The idea of creating an army out of the Indian prisoners of war (POWs) was originally
that of Mohan Singh, an Indian army officer who had decided not to join the retreating
British army in Malaya. He decided to turn to the Japanese
for help.

The Japanese had till then encouraged Indian civilians to form anti-British organisations.
Mohan Singh asked for Indian prisoners of war.

The Japanese handed over the Indian prisoners of war to Mohan Singh who tried to recruit
them into an Indian National Army. After the fall of Singapore, several POWs were ready to
join Mohan Singh. By the end of 1942, 40,000 men were ready to join the INA.

It was intended that the INA would go into action only on the invitation of the Indian
National Congress and the people of India. The move to form this army has been seen by
many as a check against the misconduct of the Japanese against Indians in South-East Asia
and as a bulwark against a possible future Japanese occupation of India.

95. Consider the following statements regarding Financial Stability and Development Council

www.insightsactivelearn.com 80
Total Marks : 200
Test-4 (Subject)
( INSTA Prelims Test Series 2021 )

1. It is chaired by RBI Governor


2. The Council can invite experts to its meeting if required.
3. It members include the heads of financial sector in India

Which of the statements given above is/are correct?


A. 3 only
B. 2 and 3 only
C. 1, 2 and 3
D. 2 only

Correct Answer : B

Answer Justification :

Financial Stability and Development Council:

The Financial Stability and Development Council (FSDC) is a non-statutory apex council under
the Ministry of Finance constituted by the Executive Order in 2010.

The objective of FSDC is to strengthen and institutionalize the mechanism for maintaining
financial stability, enhancing inter-regulatory coordination and promoting financial sector
development.

It is chaired by the Finance Minister and its members include the heads of all
Financial Sector Regulators (RBI, SEBI, PFRDA & IRDA), Finance Secretary, Secretary of
Department of Economic Affairs (DEA), Secretary of Department of Financial Services (DFS),
and Chief Economic Adviser. Hence Statement 1 is incorrect.

The Council can invite experts to its meeting if required. FSDC sub-committee is headed
by the Governor of RBI. Hence Statement 3 is correct.

96. Consider the following statements regarding Indian response to the Simon Commission
1. The Congress decided to boycott the commission.
2. Muslim League under Jinnah decided to support the government.
3. Unionists in Punjab and the Justice Party, decided not to boycott the commission.

Which of the statements given above is/are correct?


A. 1 only
B. 1 and 3 only
C. 2 and 3 only
D. 1, 2 and 3

Correct Answer : B

www.insightsactivelearn.com 81
Total Marks : 200
Test-4 (Subject)
( INSTA Prelims Test Series 2021 )

Answer Justification :

The Indian response to the Simon Commission:

The Indian response to the Simon Commission was immediate and nearly unanimous. What
angered the Indians most was the exclusion of Indians from the commission and the basic
notion behind the exclusion that foreigners would discuss and decide upon India’s fitness for
self-government. This notion was seen as a violation of the principle of self-determination, and
as a deliberate insult to the self-respect of Indians.

Congress Response

The Congress session in Madras (December 1927) meeting under the presidency of
M.A. Ansari decided to boycott the commission “at every stage and in every form”.
Meanwhile Nehru succeeded in getting a snap resolution passed at the session, declaring
complete independence as the goal of the Congress. Hence, statement 1 is correct.

Other Groups

Those who decided to support the Congress call of boycott of the Simon Commission included
the liberals of the Hindu Mahasabha and the majority faction of the Muslim League under
Jinnah. The Muslim league had two sessions in 1927 – one under Jinnah at Calcutta where
it was decided to oppose the Simon Commission, and another at Lahore under
Muhammad Shafi, who supported the government. Hence, statement 2 is incorrect.

Some others, such as the Unionists in Punjab and the Justice Party in the south,
decided not to boycott the commission.

Hence, statement 3 is correct.

97. Consider the following statements regarding Desai-Liaqat Pact


1. An equal number of persons nominated by the Congress and the League in the central
legislature
2. 50% seats should be reserved for minorities.

Which of the statements given above is/are correct?


A. 1 only
B. 2 only
C. Both 1 and 2
D. Neither 1 nor 2

Correct Answer : A

Answer Justification :

Desai-Liaqat Pact
www.insightsactivelearn.com 82
Total Marks : 200
Test-4 (Subject)
( INSTA Prelims Test Series 2021 )

Efforts continued to end the deadlock. Bhulabhai Desai, leader of the Congress Party in the
Central Legislative Assembly, met Liaqat Ali Khan, deputy leader of the Muslim League in that
Assembly, and both of them came up withthe draft proposal for the formation of an interim
government at the centre, consisting of

An equal number of persons nominated by the Congress and the League in the
central legislature. Hence Statement 1 is correct.
20% reserved seats for minorities. Hence Statement 2 is incorrect.

No settlement could be reached between the Congress and the League on these lines, but the
fact that a sort of parity between the Congress and the League was decided upon had far-
reaching consequences

98. Consider the following statements regarding Particularly Vulnerable Tribal Groups
1. They are mostly homogenous, with a small population.
2. Among the 75 listed PVTG’s the highest number are found in Odisha.

Which of the statements given above is/are correct?


A. 1 only
B. 2 only
C. Both 1 and 2
D. Neither 1 nor 2

Correct Answer : C

Answer Justification :

Both the statements are correct.

75 tribal groups have been categorized categorized by Ministry of Home Affairs as Particularly
Vulnerable Tribal Groups (PVTG)s. PVTGs reside in 18 States and UT of A&N Islands. The
Ministry of Tribal Affairs implements the Scheme of “Development of Particularly Vulnerable
Tribal Groups (PVTGs)” exclusively for them.

Under the scheme, Conservation-cum-Development (CCD)/Annual Plans are to be prepared by


each State/UT for their PVTGs based on their need assessment, which are then appraised and
approved by the Project Appraisal Committee of the Ministry. Activities for development of
PVTGs are undertaken in Sectors of Education, Health, Livelihood and Skill Development,
Agricultural Development ,Housing & Habitat, Conservation of Culture etc.

PVTGs have some basic characteristics - they are mostly homogenous, with a small
population, relatively physically isolated, absence of written language, relatively
simple technology and a slower rate of change etc.

Among the 75 listed PVTG’s the highest number are found in Odisha.

www.insightsactivelearn.com 83
Total Marks : 200
Test-4 (Subject)
( INSTA Prelims Test Series 2021 )

https://www.tribal.nic.in/pvtg.aspx

99. Consider the following statements regarding Wavell Plan


1. Hindus and Muslims were to have equal representation in executive council.
2. All members of the executive council were to be Indians.

Which of the statements given above is/are correct?


A. 1 only
B. 2 only
C. Both 1 and 2
D. Neither 1 nor 2

Correct Answer : A

Answer Justification :

Although the war in Europe came to an end in May 1945, the Japanese threat still remained.
The Conservative
government in Britain led by Churchill was keen to reach a solution on the constitutional
question in India. The viceroy, Lord Wavell was permitted to start negotiations with Indian
leaders. Congress leaders were released from jails in June 1945.

The Plan

The idea was to reconstruct the governor-general’s executive council pending the preparation
of a new constitution. For this purpose, a conference was convened by the viceroy, Lord
Wavell, at Shimla in June 1945.

The main proposals of the Wavell Plan were as follows.

● With the exception of the governor-general and the commander-in-chief, all


members of the executive council were to be Indians. Hence Statement 2 is incorrect.

● Caste Hindus and Muslims were to have equal representation. Hence Statement 1 is
correct.

● The reconstructed council was to function as an interim government within the framework
of the 1935 Act (i.e. not responsible to the Central Assembly).
● The governor-general was to exercise his veto on the advice of ministers.
● Representatives of different parties were to submit a joint list to the viceroy for nominations
to the executive council. If a joint list was not possible, then separate lists were to be

www.insightsactivelearn.com 84
Total Marks : 200
Test-4 (Subject)
( INSTA Prelims Test Series 2021 )

submitted.
● Possibilities were to be kept open for negotiations on a new constitution once the war was
finally won.

100. Toda Embroidery, sometime seen in the news, belongs to which of the following habitat?

A. Nallamala Hills
B. Nilgiri Hills
C. Ajanta Hiils
D. None of the above

Correct Answer : B

Answer Justification :

The Toda Embroidery, also locally known as "pukhoor", is an art work among the
Toda pastoral people of Nilgiris, in Tamil Nadu, made exclusively by their women. The
embroidery, which has a fine finish, appears like a woven cloth but is made with use of red and
black threads with a white cotton cloth background.

https://www.thehindu.com/news/national/tamil-nadu/toda-embroidery-duplicates-threaten-local
-artisans-livelihood/article28527229.ece

www.insightsactivelearn.com 85

You might also like